Download as pdf or txt
Download as pdf or txt
You are on page 1of 148

1

Lesion on lip (cold sore)


History:
 You are FY2 in GP
 A 28 year old lady has presented with a rash on her lips (lower lip)
 She noticed that two weeks back , it was a blistery rash
 She bought a topical cream from the medicine store but didn’t help
much
 No pain , no discharge, no bleeding , there is no lesion anywhere , and
it’s a solitary lesion
 She has a baby of 7months old
 She is married for 3years lives with husband, practice oral sex
 Partner has no similar lesion anywhere , or no discharge from private
part
 No fever no other symptoms
Task:
Talk to her and discuss management
Concern:
Will my baby gonna get it?
Examination:
Take observation, head to toe
Examine the lesion
Management:
 Greet and confirm the identity
Take focused history of the complaint
ODPARA of the lesion
Size , site , shape , color, bleeding oozing , discharge , painful painless

1
2
Associated symptoms, fever , nausea, any infection
FLAWS, DD (trauma , insect bite, infectious mononucleosis)
Take sexual history, history of safe sex , oral sex
If partner has similar rash
Do ICE
 Verbalize examination and summarize
 This lesion is called cold sore , it is caused by herpes simplex virus;
assure her it will clear up by its own within 7-10 days
 Advice
Do’s
1. Eat cool and soft food
2. Wash you hand with soap and water
3. Use antiseptic mouth wash, drink plenty of fluids
4. You can take PC for pain
5. You can use over the counter sore cream
Don't
1. Do not touch or scratch the site
2. Avoid oral sex, avoid kissing your child
3. Do not share your items with anyone
 Address concern: this is contagious your baby can have it

2
3

Chicken pox exposure in pregnant woman


History:
 You are FY2 in GP
 A 34 year old lady has come to you with some concerns
 Her 3 year old boy was diagnosed with chicken pox yesterday , her
husband took him to the GP with history of fever and rash
 Her baby is doing well now
 She is worried about her unborn child , she is 36 weeks pregnant
 She is generally fit and well , pregnancy going uneventful so far
 Has taken all antenatal heck ups
 she is immunized in childhood
 She had chicken pox in childhood; also her son is up to date with
vaccine
Task
Talk to her and address her concerns
Concern
Will my unborn baby get chicken pox?
Examination
Take observation , head to toe
Examination of tummy (verbalize , fetal heart sound)
Management
 Greet and confirm identity
Take focused history
3
4
History of son’s chicken pox
what symptoms does he have?
When did he start feeling unwell? Any rash? Any fever?
Did he go to see a Dr?
is he taking any Meds?
How is he doing now?
Have you been in contact with him since he was diagnosed?
History of exposure:
Is he living with you?
Has he been with you for the last 7days?
Have you been in contact with him?
History of chickenpox in the mom:
do you have any symptoms? Any fever? Any rash? Any headache? Did
you vomit?
History of exposure in the mom:
Did you have chickenpox as a child?
Did you have vaccination for chickenpox?
Ask about her pregnancy, hows pregnancy going so far
Red flags (HTN , GDM , pre eclampsia etc)
DESA
MAFTOSA
 Verbalize examination and summarize
 Assure her:
1. she doesn’t need to worry as she had chicken pox as a child, she is
unlikely to suffer from chicken pox again
2. This is a one-time infection and is unlikely to affect your pregnancy. If
the baby was to be affected by chickenpox.
4
5
3. The baby would be born with chickenpox and he will need treatment
but the baby will not any abnormalities because your pregnancy is
36weks. chickenpox will not have any effect on the baby as all organs
are formed, so risk of abnormalities is low.
 Investigation: Test for immunoglobulin G ab
 Advice: pregnancy related advice , food and hygiene
 Safety netting: rash. fever

Bi temporal hemianopia
History:
 You are FY2 in emergency medicine
 A 58 year old man presented with problem in his eyes
 He was brought to the hospital by her wife , she said he need to check
up his eyes
 A week back he broke the mirror of his car on left side, also there are
multiple scratches on the car , and he doesn’t know how that happen
 He find difficulty seeing objects at the corner, or difficulty in navigating
at a busy street
 No eye pain , no redness, watering or headache, no trauma in head
 He has been driving for 30 years
 He is generally fit and well
 No co morbid condition , no smoking or drinking habit
Task
Talk to him and discuss management
5
6
Concern
Is it cancerous?
Examination
Take observation, head to toe
Complete eye examination , inspection , visual acuity, visual field, light
reflex
(patient couldn't see the pin when moved from centre to the corner)
Management
 Greet and confirm identity
Take focused history
What brought you here today?
Explore the complaint, ODPARA
Why did your wife ask you to come and see us?
Why did your wife think you have a problem?
What types of accidents have you had?
Have you had any accidents at home?
Have you bumped into things that are directly in front of you?
Have you bumped into objects on your sides?
Eye questions (any pain , redness, blurry vision , watery eye, discharge,
trauma, associated symptoms etc)
Differentials : glaucoma , GCA, ARMD, optic neuritis, uveitis, retinal
detachment, pituitary adenoma, stroke , retinitis pigmentosa etc
MAFTOSA
DESA
Do ICE
 Verbalize examination and summarize

6
7
 From the history and examination we are suspecting you might have a
condition called bi temporal hemianopia, this means you have lost
vision on both side of your eyes
 Involve senior
 Urgent admission to the hospital
 Urgent call to neurologist and endocrinologist (as this condition is
caused by a gland tumour in the brain called pituitary adenoma)
 Investigation: all blood routine , chest xray , urine RE, MRI needed for
brain done after specialist approval
 Treatment: (includes medication to control hormone to shrink the tumor,
surgery or radiation) in this case since its affecting the vision surgery
procedure might needed; that involves putting a thin tube through nos
and resecting the tumor
 Address concerns: this tumour is usually non cancerous
 Advice: avoid driving

7
8

Intermittent claudication
History:
 You are FY2 in GP
 A 65 year old man has made an appointment to see you
 He has pain in both legs, mostly in his calf muscle
 It started 6 months back and becoming worse day by day
 Pain is dull, increased on exertion and relives at taking rest, it does not
go anywhere
 The pain is 4/10 on right and 5/10 on left
 He took PC but didn’t help
 No trauma, no swelling , no redness, no temperature
 He has hypertension for 15 years takes medicine for that
 He is a chronic smoker , smokes 20cigarettes a day for 30years now;
he drinks occasionally
 He loves to play golf , but can not stand any longer due to pain
Task
Talk to him and discuss management
Concern
Want to get rid of this pain Dr.
Examination
Take observations and head to toe
Examination of both legs, color change of skin of leg, checking for any
ulcers, SRLT, ABPI. Pulses, neurological examination, heart lung
examination , BMI , Q risk
8
9
Management
 Greet and confirm identity
Take focused history of leg pain
SOCRATES of pain
Associated symptoms , FLAWS
Differentials: acute limb ischemia, intermittent claudication , DVT, spinal
stenosis, diabetic neuropathy, trauma, cauda equina syndrome , problem
with bowel bladder etc
MAFTOSA
DESA
Do ICE
 Verbalize examination and Summarize
 It is the narrowing of the arteries of the legs. Usually leads to pain in the
calf, hip, buttocks, can be worse during exercise, the primary cause of it is
peripheral artery disease
 Refer to vascular surgeon to reassess your condition and advice you
about treatment like angioplasty or bypass surgery. (according to ABPI ;
if its less than .5, severe and refer urgently, >-5 but <.8 refer routinely)
 Involve senior
 Investigations: Routine bloods (FBC, RBS) + Lipid profile , Doppler
USG
 Supervised Exercise program: 2 times of supervised exercise a week
for the last 3months or exercise to point and maximal pain or exercise
30min. 3-5 times a week and or exercise till you feel the pain and rest to
recover.
 Medications: Aspirin 75mg.
 Advice
9
10
1. to stop smoking as it can cause spasm of arteries and veins
2. Stop driving and inform DVLA
3. BP monitoring
4. Follow up in 3months
 Safety netting: persistent or worsening pain, changes in color, swelling
and redness, sudden chest pain, SOB

Combined speculum examination:


History:
 You are FY2 in GP
 A 30year old has booked an appointment to talk to you
 She has been having heavy menstrual period for last 6months also pain
and discomfort in the lower tummy
 Period is regular and lasts for 7days and cycle is 28days
 LMP was 5days back
 No intermentrual bleed, no bleeding after sex
 she has one child 3years old, not using any devices, use condoms
 Last Pap smear 5years back
 MAFTOSA DESA normal findings
Task
 Talk to her and perform pelvic examination and discuss findings
examination and investigation
 Take observation and head to toe, tummy
 Setting pelvic mannikin, speculum, hand gel , gloves, split lamp
Management:
10
11
 Greet and confirm identity
take focused history of bleeding
(regularity, duration, days of circle, heavy or light, any clots , pain, LMP ,
inter menstrual bleed etc)
Ask about sexual history,
use of device,
post coital bleed,
cervical screening,
pregnancies,
ask DD, (hyperthyroidism, PCOS, PID)
associated symptoms anemia
Past history,
ICE
 Do PPCCE, examine the pelvis as you were taught
 Findings cervix reddish with clots
 Explain she is having menorrhagia, this heavy bleeding can be caused
by many reason like fibroid
 Arrange investigation, cervical smear, US scan, clotting factor etc
 If all test comes normal, and if bleeding continues we can offer mirena
coil
 Safety netting

Eye examination:
History:

11
12
 You are FY2 in GP clinic
 A 16-year-old girl presented with bilateral blurring of vision, redness,
and pain for 2days
 She uses contact lens which were a cosmetic single use lens.
 She slept with wearing lens two days ago at night.
 After waking up in morning she tried to remove it and developed pain
and redness in right eye.
 Gritty sensation present.
 Discharge is present on right eye.
 No photo phobia. No neck stiffness. No rash, No headache.
 MAFTOSA DESA normal
Concern:
What happened to me?
Examination and investigation:
Examination of the eye: Examine the eyelids, conjunctiva, cornea, and
pupils. Check for redness, discharge, swelling of eyelids, any foreign body,
check the pattern of redness.
Fundoscopy, Visual Acuity, Check Eye pressure, Visual acuity Right eye
6/18, left eye 6/36.
Routine blood, inflammatory marker
Management:
 Greet and confirm identity.
Explore Pain,
SOCRATES,and
Discharge (TRAC)
Rule out differentials,

12
13
Foreign Body Sensation, Discharge, Itchiness, Photo-phobia, Headache,
or any other associated symptoms.
Explore P2, P3, MAFTOSA, Driving, Psycho-social.
History of Contact lens use,
any trauma,
any chemical exposure,
any medical condition associated with causes of red eye (Atopy,
autoimmune disorders etc), Acute Glaucoma , Scleritis and
Episcleritis/,Conjunctivitis
 Do ICE,
 verbalize examination and Explain diagnosis, we are suspecting Lens
induced keratitis which is the inflammation of the cornea caused by
repetitive or unhygienic use of the contact lenses. It is a serious and
potentially sight-threatening condition and needs to be assessed by the
ophthalmologist immediately.
 We will need to refer the patient immediately as there is a risk of further
visual loss.
 Refer the patient to the emergency eye service for same-day
assessment by an ophthalmologist.
 Investigation: They will examine your eyes again and might take swab
sample from your eyes to see if the inflammation is being caused by any
bugs. Slit lamp examination for contact lens keratitis
 They will start you one eye drops based on the findings, if infection is
suspected you will be started on antibiotic eye drops.
 Advice
1. the patient not to drive and if needed arrange ambulance for the patient.

13
14
2. Reassure the patient that if treated immediately the visual loss might
improve with treatment.

DKA Refusing admission


History:
 You are FY2 in emergency department
 35 year old lady has come to the hospital with abdominal pain and
vomiting

 She is a single mom. She has a wedding in 5 days. She has to attend.
She doesn’t want to be admitted and want to go home.

 She was lying on hospital bed with IV fluids.


 she has DM type 1 for 5 years .
 She was on 20 units long acting insulin which she takesin the morning.
 For the last 5 days she has been busy with work and herchild so she did
not take the insulin.

 no other symptoms, patient doesn’t want to stay in thehospital, she


needs to bring her child from nursery too

Task:
Take focused history, discuss diagnosis and management.
Concern:
What are they going to do for me in the hospital?
doctor I have my wedding needs preparations also I need bring
my daughter from the nursery
14
15
Examination and investigation:
take observations, examine tummy
Blood (RBS/ ESR, CRP/ cholesterol/ ABG/ KFT),
Urine (dipstick culture ketone bodies), chest X-ray for infection,
Erect abdominal X-ray, RBS, ABG (metabolic acidosis)
Management:
 greet and confirm identity

explore presenting complaints and take full history of diabetes

for how long,

medication,

if it’s controlled,

complications,

check ups,

any challenges, what happened today

exclude red flags

infection,

DESA,

MAFTOSA

 Do ICE,

 verbalize examination and investigation and explain what is DKA; it’s a


serious condition if not treated can be life threatening

 tell the patient he needs admission, if patient refuses


15
16
 use refusal approach

 Treatment in the hospital: insulin via drip, fluid to reduce dehydration

 Address concern of the patient, ask for the reason of refusal and
give solutions, if she says she has wedding preparations tell her can
anyone give her a hand for thepreparations , also ask for someone else
to bring her child

 Explain she needs to stay at the hospital now as her condition might
get worse which will require more aggressive treatment

SIMMAN
History:
 You are FY2 in emergency department
 A 55year old man has come with pain in his right leg for 5hours
 Started suddenly,severe,sharp,non radiating pain,score 9/10
 No associated symptoms,injury or breathing problem,swelling or
bleeding,but a thumping feeling on chest
 This is the very first time he has been experiencing something likethat
 He is hypertensive and takes medicine for that and well controlled
 A smoker for 20years smoke 15to 20 cigarettes per day
 He can’t feel his leg as it feels like numb
Task:
16
17
Assess the patient and manage accordingly
Examination and Investigations:
Observations
Temp 37,
HR 92
RR 18
BP 100/72
SPO2 98.Right leg is painful on palpitation
radial pulse is irregular,dorsalis pedis pulse is absent on the right side,
right leg pale white colored
ECG shows Atrial fibrillation
Routine blood, ECG,chest xray, Bleeding and clotting profile,
Lipidprofile, d-dimer,cardiac enzyme
Management:
 Greet and confirm identity,take universal precaution and introduce
yourself
 Take focused history of pain,associated symptoms,perform ABCDE
protocol
 Do full leg examination(look,feel,touch,tap,movement
Explain the diagnosis,acute limb ischemia secondary to AF as you have
found pulselessness,painful,paralysis,pallor,paresthesia,cold limb
 Do two large bore cannula in arms, give morphine for the pain, give
oxygen, raise the leg, and low molecular weight heparin
 Urgent admission and call to vascular surgery for
urgent angiography and embolectomy to save the limb
 They will do Doppler ultrasound to confirm the diagnosis

17
18
 Also call to cardiology team for the management of heart rhythm and
rate

18
Gender dysphoria;
History:
 You are FY2 in general practice
 A 16 year old girl has booked an appointment to talk , she is on video
call
 The opening sentence is she wants to change her sex, she is biologically
a girl but wants to become a male

 She currently living as a female


 She has talked about this with her family members, they are supportive
 She has been feeling that she is in a wrong body since 8 years of age,

 She has chosen a name for her Stuart she wants her to be called by this
name

 She know something about this sex change operation as she has been
started going to the LGBT foundation already

 She goes to school, has friends there ; they are supportive too

 She doesn’t have any partner currently, she is not bullied or being forced
by anything or anyone

 She doesn’t have any medical condition, do not smoke or drink or take
drugs

12
 Wants to start transitioning, wants to know what are her options and the
process.

 She's not very confident about whether she wants to do it or not.


Task:
Talk to her , address her concerns
Concerns:
dr how can I become a male?
Am I doing the right thing?
Management:
 Greet and confirm identity, start with video approach

take past history of sexual orientation

when did she notice she is in the wrong body?

D o e s s h e identify herself as male of female?

Is she biologically male of female?

Did she discuss to anyone regarding her wish?

Was anything happened to make her feel like that)

family and friends , support group

sexual life: not sexually active


MMA: none

12
 Explain, she is having gender dysphoria which is a feeling of unease a
person experiences due to mismatch in their biological sex or gender

 refer to the gender identity clinic for further assessment ( if its sure she
doesn’t have any mental health condition)

 they will help her with support and available options which are surgery,
hormone therapy , speech , language therapy

 Appreciate her to have more information from the LGBT foundation

 Address concern, its completely natural to have questions and


doubts when you are making such decision about your identity. What
matters most is how you feel inside and what will bring you the most
happiness and fulfillment.

 Advice her for discussing with family before taking decision

nocturnal enuresis

12
History:
 You are FY2 in GP
 A mother of 4year old kid has come to see you
 Her son is wetting bed every night , he is dry in the day time , he was
never dry at night
 He is active and playful, no stress at home , started going nursery for few
months , everything going well
 He is potty trained, he feels guilty in the morning , but she never scolds
him
 BIRDDD , MAF are fine
 No other problem , he is generally fit and well
Task:
Talk to mother address her concern
Concern:
What shall I do?
Examination and investigation:
Take observation and routine test , urine dipstick
Management:
 Greet and confirm identity
Take focused history of bed wetting ,
was he ever dry at night ,
what about day time ,
any specific time of the night he wet the bed,
does he use toilet before going bed,
does he drinks etc
BIRDDD,
12
MAF
fever , flu, tummy pain , psycho social , any impact on life .,
any stress etc
 Verbalize examination and assure mother its normal till age 5
 Advice
1. encourage him going toilet before going bed ,
2. not to drink water or fizzy drink before going bed
3. Talk to child nicely , do not scold him ,
4. she can do reward system
5. We will follow up until age 5, if he still wets bed we will send him to
enuresis clinic
 Follow up

Prescription
History:
 you are FY2 in in pediatric department
 A 6 year old child is diagnosed with peritonsillar abscess
 Her weight is 22 kg
 Allergic history itchiness in clarithromycin
 No other regular medication to prescribe
Task

12
Prescribe phenoxymethylpenicillin , metronidazole , Paracetamol and
calculate amount of fluid and prescribe in the fluid chart
Management:
 greet the examiner
 Setting pen , prescription, calculator, BNF, two different stickers for
different form of prescription (fluid sheet andtreatment sheet )
 Prescribe medication from BNF, check BNF for the does
 As no IV phenoxymethylpenicillin , change it to benzyl penicillin IV and
metronidazole IV
 prescribe paracetamol in as required
 Calculate fluid and fill up the fluid chart ( as you w e r e taught in
the academy)

Shingles

History:
 You are FY2 in general practice
 A 55 year old man has presented with chest pain on right side which has
been getting worse for past 1 week, it’s gradual in onset, dull in character,
radiating to the back, he tried paracetamol but didn’t help

12
 He is generally fit and well, no past medical condition, medication history,
no allergies
 He also developed a rash on the right side, not itching but extremely
painful, no discharge, bleeding , it’s getting bigger
 He lives with his grandson
 No smoking or drinking habit
Task:
Talk to the patient, assess him and discuss management
Concerns:
Is it heart attack?
Will my grandson will be affected too?
Examination and investigations:
Take observations and do general physical examination, examine the rash ,
back
routine blood, ECG, chest X-RAY, antibody for chicken pox
Management:
 Greet and confirm identity
Take history
SOCRATES
Site-right chest
Onset-gradual
Character -dull
Radiation to the back
Taken PC - didn’t help
Associated factor- none
Fever- no
12
Rash other places on body- no
MMA- none
differentials (MI, Pericarditis, pulmonary embolism)
childhood history o f chicken pox- had history of chicken pox
Contact history- none
 Verbalize examination and summarize
 it’s a shingles, due to reactivation of previous chicken pox that hide in
your body.
 It can be passed to other people especially who are not immunized, or
who never had it, pregnant women and immunocompromised patients.
 SEND HOME WITH TREATMENT AND ADVICE
 It takes 4 weeks for the rash to heal
 Treatment- anti viral for 7 days, and Gabapentine for neuropathic pain
 Advice
1.to keep the rash clean and dry, wear loose fitting cloths
2.Put ice packs, do not cover the rash or put dressing
Offer vaccination, address concern
 Safety netting

12
Insomnia
History:
 you are FY2 in general practice
 A 37 year old man has come to you with some concerns, he has been
having sleeping problem for last 2 months
 He goes bed early but takes time to fall asleep
 Problem started since his divorce, he feels guilty
 he only has sleep 3/4 hours per day , feels tired
 No medical condition, OSA, or FLAWS
 His mood is 4, lost interest in everything , planning to leave the job , he
has lack of energy
 Living alone , didn’t talk to family and friends
 Started drinking alcohol a lot, and smoking
 No suicidal thoughts, hallucinations
 While explaining depression he was surprised saying he thought
depression means suicide
Task:
Talk to the patient, assess him and discuss management
Examination and Investigations
Take observations and general physical examination
Routine blood tests
Management:
 greet and confirm identity
explore sleeping problem,
ODPARA,
sleep hygiene,
12
does he wakes up in the middle of the night or not,
any day time naps,
do differentials and systemic review
assess mood,
ask two core symptoms of depression,
past incident,
ask about support, friends and family
Do MAFTOSA and DESA
 Exclude suicidal thoughts and hallucinations
 Verbalize examination and summarize he is having depression after
divorce
 immediate referral to psychiatrist
 Prescribe anti depressants and offer CBT
 Tell him to talk with friends and family, to open up
 advice
1. cut down alcohol consumption
2. smoking cessation
3. offer crisis card , and tell him to call anytime to the number
 Safety netting

12
Blocked cannula:
History:
 You are FY2 in pediatric department
 A 14year old boy has been admitted due to chest infection
 He was prescribed IV antibiotic, received 2days dose ; 3days left, then his
cannula got blocked, it was tried 4times but still failed , as veins are not
visible enough
 Mother became very angry as she things her son is in pain , and could not
tell
 Her son has cerebral palsy , can not walk or talk , goes to special schools
 Her son developed tachycardia and fever
 He got repeated infection and admitted hospital before several times
 Otherwise fine
Task:
Talk to mother address her concern
Concern:
Why you are giving pain to my child?
He doesn’t need antibiotic , he is doing fine
Why he is having repeated infection?
Management
 Greet and confirm identity, confirm relationship
Ask present condition ,
hows her child ,
why he was admitted ,
how is he doing now ,
any problem
12
Paraphrase the scenario , I understand you are upset,
ask why she doesn’t want to insert the cannula,
does she understand what will happen if cannula is not inserted etc
Ask about cerebral palsy ,
since when , how she is coping up ,
what are the challenges mostly,
any other disease etc
 Now show empathy and explain why he needs cannula , its important to
receive the antibiotic , for the infection
 Give solution , we can put numbing gel on his hand , and can call expert
 Its distressing for both of you but inserting cannula is also essential
 He is having infection repeatedly cause his immune system is not strong
enough also due to his neck posture , sometimes food get stuck in the air
pipe can cause infection
 Address concern and show empathy
 Ask for any support she needs

12
PCOS
History:
 You are FY2 general practice
 A 25 year old woman has come for the test result report
 She came 2week back with the complaint of acne and irregular period
 Last menstrual period was 2months back
 She has been gaining weight for last 1year and also noticed somefacial
hair,
 She is normally fit and well, not sexually active
 She has no medical condition and not on any medication
Task:
Talk to the patient and explain test result and management
Concern:
It it treatable?
I do not want to take COCP
Examination:
Acne, head to toe, observations, BMI
Test result showed high LH , FSH normal, LH:FSH 3:1 ,BMI 32
Management:
 Greet and confirm identity
Paraphrase the scenario,
take focused history,
present condition ofacne , facial hair, weight, period, pills ,
ask psycho social,
MAFTOSA ,
DESA
12
 Do ICE, explain test result , we are suspecting PCOS which is a
condition of varies in which the ovaries produce abnormal amounts of
androgen male’s hormones. And there be many cysts in the ovaries. It
can cause many symptoms like periods irregularities, mood swings.
 Prescribe cyclical progesterone for period or mirena coil ( as she
declines COCP); if she declines all refer to specialist
 Follow up for a month: to see menstrual irregularities; if present
withdrawal bleeding with medroxyprogesterone for 14 days will be
needed and USG to see endometrial thickness
 Advice:
1. Tell her to lose weight first,
2. refer to dietitian, and also regular exercise
3. For facial hair she can do waxing, shaving or laser
4. For acne refer to dermatologist
 Follow up
 Safety netting

Lithium toxicity

History:
12
 You are FY2 in emergency department
 A 63 year old man was brought to the hospital by his daughter as he was
confused since morning
 Also peeing a lot and seems tired
 Daughter is anxious, when you asked he is diagnosed with bipolar mood
disorder for which he has been taking lithium for 6months
 He doesn’t have other co morbidity, like DM , HITN , Thyroid problem
 Retired and Lives with daughter , daughter takes care for him
 No smoking or drinking habit
 No history of dementia, fever, infection, stroke, trauma or fall
Task:
Talk to the daughter , assess him and do management
Concern:
What happened to my father?
Examination:
Take observation, head to toe,
Neurological examination, MMSE
Management:
 Greet and confirm identity
Take history
Access the confusion state
Since when?
what do you mean by confused ?
Exclude dementia by asking: has he been forgetful recently?
Does he struggle to find words ?
Getting lost in familiar places ?
12
DDs for confusion :
1) UTI : ask about urinary symptoms : going yo the loo more often ? Any pain
passing urine ? Any fever ?
2) Diabetes : Drinking
more than usual lately ?
3) Stroke : Any weakness anywhere in his body ?
4) Head injury :
Any recent falls ?
MMA: when lithium is mentioned ask about
Why prescribed ?
Since when ?
Taking as prescribed ?
Going Follow up appointments?
Dose change recently ?
Other side effects of lithium toxicity : Nausea & vomiting? Heart racing ?
Tremors ?
Any other medications ? (to exclude interaction)
psycho social of father and daughter:
does he live alone ?
Any one caring for him other than yourself ?
Is he independent with daily activities?
Who else do you care for ?
Do you find it difficult to care for him?
Any help we can offer ?
 Do ICE
 Explain that you suspect that his confusion is due to lithium toxicity ,
12
that’s when the lithium level in the blood increases over the recommended
dose ( mention cause if said by daughter , if not then say we need to do
further investigations to see why this happened )
 Admit the patient right away and stop lithium
 Involve senior
 Investigation: FBC, RFT, TFT, U&E, Lithium blood level , ECG
 Management
1. IV fluids
2. Recheck lithium level every 6-12 hours
3. Might need heamodialysis if level is too high
 Refer to psychiatrist for bipolar reassessment and dose adjustment.
 Advice
1. continuous monitor / follow up appointments
2. no OTC NSAIDs and to maintain hydration by fluid intake
3. Advice not to stop lithium suddenly and always seek medical advice first .
 Offer support and carer if needed
 Safety net about deterioration , neurological manifestations ( other signs
of toxicity)

BBN:
History:
12
 You are FY2 in emergency department
 A 77 years old man who has collapsed at home andbrought to the
hospital by an ambulance.
 Patient’s wife has come to see her husband
 she was watching TV with his husband when he suddenly collapsed. She
called the ambulance and he was brought to the hospital. She came to the
hospital byher own.
 The neurosurgeon has assessed him and have classified the condition as
terminal as they felt an operation wouldn’t resolve the situation.
 The CT scan of the head was done and showed massive intracranial
hemorrhage. The neurosurgeon believe it is berry’s aneurysm.
 Patient is lying unconscious and breathing independently.
 He is hypertensive and on ramipril, diabetic and has stable angina.
 Non smoker, non alcoholic
Task:
 Talk to the patient’s wife, tell her about patient’s condition and address
her concern
Concern:
 Are you going to send him to ICU?
 How much time does have left?
 Would he need surgery?
Management:
 Introduce yourself, greet and confirm identity and relationships
Assess knowledge
take history about the incident, (before-during-after)
Ask about bleeding disorder,
12
medical condition,
medication history,
blood thinners,
DESA
 Do breaking bad approach, ‘your husband was brought to the hospital by
an ambulance unconscious. Unfortunately, He is still unconscious and we
have done
 CT scan on his head But, the results were not what weare hoping for.
 explain the massive haemorrhage and what specialist has taken
decision.
 Offer support.
 Explain end of life care
 Address concern
1. Unfortunately, ICU is where we putpatients who will recover. It won’t be
beneficial to him.
2. It is difficult to say how much time he has left but I willspeak to
surgeons if they can give us a time frame.
3. explain her the possible cause of this , like hypertension explain why
surgery is not possible
 Talk in a very calm voice and comfort her, answer all herquestions.

12
post op arthroplasty

History:
 You are FY2 in orthopaedic department.
 A 79 year old woman has been admitted to the hospital. She had a hemi-
arthroplasty 4 days back because of fracture in the neck of femur. She
has not passed any stools since she was admitted.
 She is on co-codamol for pain. Pain Is well controlled and she is stable.
 She had no constipation before admission. Has been unable to empty her
bowel for 7days now. Before fracture she used to open bowel daily.
 She was also fully mobile before the fall.
 She is not on any medication and can pass wind.
 She takes a lot of fluids and fiber.
 No previous history of allergies and no significant medical history.
 She wants to go home , lives alone no ones thereby
Task:
Please talk to her and discuss management
Examination and investigation:
Take observations , abdomen and back passageExamine wound site
Management:
 Greet and confirm identity
build rapport
Ask about operation,
any complications, pain , symptoms,
about hospital stay and care
Explore Constipation,
12
ODPARA,
rest of GIT symptoms such as tummy pain, diarrhea, vomiting, fever, nausea
Differentials
DESA,
MAFTOSA,
 Ask concern , idea
 Verbalize examination and summarize,
 explain the pain killer you are taking could be the cause of your
constipation
 open BNF and show her
 Reassure and Advice
1. take fruits and vegetables as they are enriched with fibers , take Plenty of
fluids
2. Advice Mobilization, Stop the co-codamol and assess the pain to
prescribe another one like paracetamol
 Offer Laxatives
 involve multidisciplinary team , physiotherapist to star appropriate
mobility and occupational therapist to assess her living situation
and make adjustments.
 Offer if she needs any carer
 Safety netting

12
learning disability
History:
 You are FY2 in general practice
 A 40 year old man has made an appointment to see GP, he had a fit
yesterday
 This is 3rd time happened; he was diagnosed epilepsy and was admitted
in the hospital, he was discharged on NA valproate twice a day
 Patient had learning disability and he was not taking medicine as
prescribed
 Yesterday he was partying and the music and alcohol triggered and he
had a fit
 He was shaking and lost his consciousness for few moment
 Before the fit he didn’t experience any symptoms, any visual or
auditory experience, no headache, no vomiting
 He didn’t soil himself or bite his tongue
 After the fit he wasn’t drowsy, no abnormalities, no
headache or confusion,no paralysis or memory loss
 He drinks every weekend and go to pub and party
 He had no fever,or any medical conditions
 Family history of epilepsy present
 No Past medical or medication history
Task:
Talk to the patient, take focused history, discuss Management
Concerns:
When I can drink?
Can I drink a little every week?
12
Examination and investigations:
Take observation and do general physical examinationincluding
neurological examination
Routine blood,urea and electrolytes, urine, ECG
Management:
 Greet and confirm identity
 Paraphrase scenario
Take history
Explore Fit,ask questions of before, during, after of the fit: was shaking and
lost his consciousness for few moment
Previous hospital stay/ treatment/ investigation:
Medication: Na valproate twice daily
Compliance: he forgot to take, learning disability present
Family history: epilepsy
MMA: epilepsy
DESA: drinks every week
 Tell him he got an attack , as he was not taking medicine
 As he has learning disability don’t use medical jargon, explain everything
in simple words, or ask him if you will write it one paper
 Advice:
1. Insist him of taking medicine, and give a reminder on phone or someone
at home who will tell him to take medicine
2. wear epilepsy bracelet/band
3. Ensure he is not driving
4. Avoid drinking alcohol, loud music, lashing lights as these will trigger the
attack; if he insists to drink then explain why drinking is harmful for him
12
 Refer to fit clinic and offer crisis card if he doesn’t have
 Safety netting of stroke,TIA

Prescription
History:
 You are FY2 in AMU
 A 83 year old man has been admitted in the hospital with upper
respiratory tract infection
 he was prescribed doxycycline 100mg BD

 he has developed non valvular atrial fibrillation

 he is also hypertensive and on atenolol

Task:

 Prescribe apixaban and atenolol


 Setting BNF prescription, black pen etc
Management:
 Greet the examiner and start writing prescription
 Check doses of apixaban from the BNF (criteria one prescribe
apixaban 2.5 mg BD oral

 Put start date , and continuous


12
 Prescribe atenolol as it

Teaching
History:
 You are FY2 in a surgical unit.
 a 5th year medical student who attended a lecture on groin and swelling
examination.
 He attended the lecture but he has never seen one being done before.
Task:
 Teach the medical student inguinal scrotal examination.
SET UP: male pelvic mannikin. Torch, Paper.
Question asked by the student:
What’s the difference between direct and indirect inguinal hernias?
What are the other possible causes of groin lumps?
When do you do surgeries?
Teaching method:
 Greet and introduce yourself
how are you doing?
How is medical school?
Do you have any exams soon?
I understand that you want to learn to learn how to perform inguinal scrotal
exam. Have you seen any one before?
12
What do you know about it?
have you done one before?
 Explain the Reason: this examination can be done for various reasons
but the most common indications are groin swelling and pain in the groin.
 Steps:
 Show The important inguinoscrotal landmarks
The inguinoscrotal landmarks are: ASIS, pubic tubercle, inguinal ligament,
inguinal canal, deep inguinal ring, superficial inguinal ring. Mid inguinal point
 Explain Consent, positioning, :
Inspection:
 examining/ inspecting the lower abdomen, groin, penis and scrotum
 check for DRSSS (discharge, redness, sinuses, scars, odor changes)
Cough impulse: to exaggerate the hernia or any other swelling that may
have a cough impulse such as varicoceles
Palpation:
 palpate the important anatomical land markers.
 Check consistency (hard, soft). compare temperature to the thigh.
 Check for tenderness. if there is swelling they will ask you to reduce it.
Scrotal examination:
 check size, surface and sides. 4 corners to get above the mass. Check if
attached to skin or attached to testis
Special tests:
 cough impulse
 occlusion test
 prehn’s sign
transillumination
12
Auscultation: auscultate the swelling for any bowel sounds
To complete exam: PR examination and lymph nodes examination
 At last thank the patient, ask him to dress up, remove gloves and discard
into clinical waste, wash hands
 explain findings to the patient.
Tell the patient of your net plan following

Teaching:
History:
 You are FY2 in emergency department
 A young lady brought her 5 years old son to the hospital after anaphylaxis
 She went to a restaurant and told them her child h a s p e a n u t allergy
but they gave him peanut. This is the 2nd time he developed anaphylaxis.
The first time was when he went to his friend’s birthday party and he was
given food with peanut.
 He was treated that time and was prescribed epipen.
 She doesn’t know how to use epipen now she wants know the use of
epipen.
Task:
Talk to the mom , address her concern and teach her how to use the epipen
In the cubicle there are two epipens, (Dami), and also one has given in his
nursery
12
Concern:
What If she doesn’t respond to it first time. Should I use it again?
what if I use the PEN and it’s not anaphylaxis?
Shall I need to call ambulance if he become alright after the shot?
Management:
 Greet and confirm identity
Explore anaphylaxis,
take history of the incident and explore symptoms
do head to toe,
BIRDDD,
MAF (take focused and shorthistory)
 assess knowledge about the e pipen, explain i t ' s p a r t s .
 Explain about the signs and symptoms of anaphylaxis (swelling of throat,
lips, eyes, difficult breathing)
 Teach epipen as you were taught
 Address concerns:
1. Use epipen when signs/symptoms of anaphylaxis and call ambulance
immediately
2. Explain she can give epipen even if she is not sure if either he is having
signs of anaphylaxis or not
3. It is not going to be harmful for the kid even if you give it to him and later
realize it is not anaphylaxis. He might experience some heart racing, etc
but it would settle in 15 to 20 minutes.
4. Always have 2 epipen, use another one in 5 minutes if ambulance doesn't
arrive
 You still need to call ambulance, after using epipen to your son
12
 Avoid triggers and advice about trigger diary
 Offer support
 Refer the patient to the allergy clinic. So that we can know the cause of the
reaction and a diagnosis can be made.
 Give the child a bracelet to wear
 Safety netting

12
48

Falls
History
 You are FY2 in emergency department
 A 68 year old lady has presented with some concerns
 She has been having fall after dizziness for 4times while trying to
change the position
 The last time happened in the emergency room while she standing up
from the sitting position
 During the fall, she fell and had a large bruise on her thigh but no
injury to her head.
 she had light hotheadedness before the fall and change of posture
 Patient is very talkative and has remembered all the episodes of fall
in last one month
 After fall she didn’t lose her consciousness, no confusion
 She came with her daughter
 she also had previous history of mild stroke 1 yr ago now she is fine
and macular degeneration eye prob as well. She missed to attend
F/U for the eye prob she said.
 no history or recent infection or epilepsy, or seizure, no paralysis of
limbs, no ear problem, no DM
 She has been diagnosed with hypertension for 10years and on
amlodipine, Ramipril and clopidogrel
 She is taking the medicine regularly as prescribed 4weeks back her
GP has done some changes to the medicines as she can recall
48
49

 Diet is healthy and she is active as well


 She is Retired teacher and live with her husband
 No history of smoking and alcohol drinking
Task:
Talk to her , address her concerns and discuss management
Concern
Why im falling repeatedly?
Examination and investigations:
 Take observation and head to toe
 Neurological examination , heart lung examinations
 Examine the BP while lying down and standing position (the two
differences should be more than 20systolic and 10diastolic)
Management
 Greet and confirm identity
Take focused history of falls
before during after
about last time,
any injury
confusion /dizziness
nausea, vomiting, blurry vision, paralysis of limbs
MMA
Family history
any recent change of lifestyle or medication
any recent infection
bowel bladder habit
 Do ICE
 verbalize examination and summarize
49
50

 you are having a condition we call it postural hypo tension; in which


your blood pressure suddenly falls with changing posture/position
(from sitting to standing position), tell her in her case we are
suspecting the cause can be changing in medicine
 Admit the patient for observation
 involve senior
 Investigation: Routine blood test, exclude any infection ,
Inflammatory markers, RBS, electrolytes, creatinine, urine R/E,
 Refer to heart specialist for further management and reviewing
the medications
 Advice
1. changing posture , like getting up slowly, avoid sudden moves and
posture changes
2. wear stocking or support on thighs and legs
3. avoid standing for long period
 Refer to physiotherapist and occupational therapist as well
 Safety netting for stroke

50
51

TIA:
History:
 You are FY2 in emergency department
 a 65 year old lady was brought to the hospital by her husband due
to weakness of one side of body.
 she had slurring of speech and l i m b w e a k n e s s f e w
hours back
 husband immediately called 999 and admitted wife to hospital.
 Symptoms eventually went away within 15 min
 DESA is unremarkable, both trying to live healthy life
 Don't know if wife has family history of heart problems orstroke.No
previous GP consults.
 No one has talked to him what TIA is and what caused it.
 husband was surprised when mentioned it was ministroke and
anxious when mentioned complication could lead to stroke.
 She is scheduled to visit a TIA clinic tomorrow.
 Husband is asking if she needs to take maintenance medications for
prevention and what to expect when they visit the clinic tomorrow.
Task:
Talk to the husband and address concern
Concern:
Is it a stroke?
How to prevent it?
Management:

 Greet and confirm identity


Paraphrase the scenario
51
52

take focused history of the incident (before during after )

Ask differentials (stroke, migraine aura, seizures, hypoglycemia,


hypertensive crisis, syncope, peripheral neuropathy

Ask FLAWS, headache, rashes, flu like symptoms , embolism symptoms

Ask MAFTOSA and DESA


Do ICE

 explain what is a mini stroke and possible cause for it, ( usually
develops if there is a blockage of blood supply to the brain. It usually
doesn’t leave any permanent damage)

 refer to TIA clinic tomorrow, for further assessment and


management

 Medicine : Aspirin with clopidogrel, Statin

 Ambulatory BP monitoring to confirm Hypertention to start anti


hypertensive

 Advice lifestyle modification, measuring BP, BMI, DESA advice


 Do not drive for at least 1 month
 Safety netting

52
53

Ingrown nail
History:
 You are FY2 in university clinic
 A 20year old university student had an injury to his big toe few days
back
 He came with painful swelling on right big toe
 Its red and hot extremely painful, lateral border
 He asked for painkiller and antibiotic
 No medical condition
 No smoking or drinking habit
 Diet normal
Task:
 Talk to him and discuss management
Examination:
 Take observation, head to toe and examine the big toe
Management:
 Greet and confirm identity
Take focused history
ODPARA of the symptoms
Associated symptoms: fever, trauma, discharge, pus, bleeding,
numbness etc
DD: trauma, gout, fungal infection , ingrown toe nail, foreign body
MAFTOSA: none
DESA: normal
 Do ICE
53
54

 Verbalize examination and summarize


 From the history the examination it looks like an ingrown toe nail, it’s
a common problem where nail grows into the toe and condition
becomes severely painful
 Treatment: Prescribe painkiller and topical fusidic acid
 Advice
do
1. Soak the foot in warm salty water to help soften the skin around the
toe and reduce the chances of infection
2. Keep the foot dry for the rest of the day
3. Wear wide , comfortable shoes and sandles
Don’t
4. Do not cut the toe nail, leave it to grow out
5. Do not pick at the toe or toe nail
6. Do not wear tight or pointy shoe
 See doctor
1. If treatment at home not helping
2. Toe is too painful with swollen pus coming out
3. High temperature , you feel hot and shivery
4. GP can give antibiotic if infected and refer to Podiatrist (foot
specialist) if no improvement in 3days
5. Podiatrist will cut the part of nail or removing the whole nail

54
55

Lumps in vagina:
History:
 You are FY2 in GP
 A 15 year old girl has come with some personal problem and feels
very shy to open up
 She has lumps in the vagina, its cauliflower like soft lumps
 No discharge, no pain , no fever, no lumps anywhere else, but those
looks very ugly
 She is worried thinking if these are serious like STI
 She is sexually active , partner is her school trusty, doesn’t practice
safe sex
 Her period is regular, LMP 1 week back
55
56

 After a while it turns out she was abused by him , she is not supposed
to discuss with anyone not even her parents
Task:
Talk to her and discuss management
Concern:
Is is something serious?
don’t tell my parents
Examination:
Take observation
Examination of tummy and genitals
Management:
 Greet and confirm identity
Take focused history of the lumps
ODPARA
Since when
How many
Anywhere else in the body or not
Does it bleed , itchy, painful, painless
Discharge
DD: syphilis, herpes , warts, fungal infection
FLAWS: negative
MMA: none
P4: LMP 1 week back , regular
Sexual history: active; no safe sex
Ask about the partner’s age
Is it mutual relationship or under any force
 Do ICE
56
57

 Verbalize and summarize, build good rapport and offer confidentiality


 Explain these are seems to be genital warts this is a sexually
transmitted infection
 Usually it doesn’t need any treatment, disappears within 6 months
 Refer to GUM clinic for further check ups
 Treatment: topical ointment can be given
 Involve senior
 Contact school safeguarding lead or designated authorities
 If the situation is serious also inform police
 Advice the girl to talk to her parents , and discuss the matter
 Offer support and any help she needs
 There is NSPCC or child line in UK to protect against any sort of
cruelty on children

57
58

Simman
History:
 You are FY2 in obstetrics and gynaecology department
 a 30 year old lady has undergone Caesarean section 8 hours
back
 She is having severe pain in the operation site
 while examination you will fine wound site soaked in blood
 No fever , vitals normal
 On the desk patient information written with drug chart
 She received dia morphine 2 hour back
 Anti emetic also given
 She already have a cannula on the hand
 All examination findings came normal
 Patient has no allergy to medication
 Urine output normal
Task:
Talk to the patient, assess her and address her concern and discuss
management with her.
Concern:
Patient insisted not taking paracetamol, asking for a stronger pain killer
like morphine , even if you try to make her understand
Examination
ABCDE approach (vitals all normal)
Management:
 Greet and confirm identity
 introduce yourself and paraphrase the scenario
58
59

 check the monitor and patient information


 do PPCCE and start with ABCDE approach
 Patient has received diamorphine 2 hours back
 change the dressing , follow pain ladder and give herpain killers
 Don’t give morphine/diamorphine as she has already received that
 Give paracetamol
involve senior to reevaluate as the wound site is soaked

Suicidal attempt:
History
 you are FY2 in psychiatry
 A 26 year old woman has presented to the emergency department
following paracetamol overdose
 She has taken 16 tablets with water at a time after a fight with her
boyfriend
 Her boyfriend wanted to break up with her
 She has done it twice before
 Her boyfriend brought her to the hospital, she denies to suicidal
attempts
 Primary assessment and treatment has been done, her PC level was
below treatment line and doctors has declared her fit for discharge
 She was otherwise fit and well, no medical condition or allergy
59
60

 She was referred to psychiatry


 Her mood was 5/10
 She has contact with family, they are aware of her suicidal thoughts
Task
 Talk to her and discuss management
Management
 Greet and confirm identity
Take focused history of suicidal attempts
(when, where, how, (before during and after the attempt questions) did
she write any notes?
did she locked the door?
does she have this thoughts sometimes?
is it the first time?
ask about past attempts?
was she brought hospital?
how she was treated?
MCFAMISH, planning, any guilt or regret?
does she wants treatment?
any mental health condition in family ?
any trauma?
psycho social support ,
MAFTOSA
Ask the effects of suicidal attempts in life, will she do it ever again? Etc
 Discuss with senior
 Advice CBT
 Educate her about the risk of self harm, also how it can affect her
family.
60
61

 If she says she feels guilty, won’t do it again discharge her home

Chest infection in homeless:


History
 You are FY2 in medicine
 A 34 year old man has presented with SOB and cough for 3weeks
 Cough is productive, scanty and white in color
 He also had sudden start of fever for 3/4 days
 His GP prescribed amoxicillin 1 week back but didn’t help
 No other associated symptoms, no weight loss , headache or rashes,
no trauma
 No past medical history, no asthma , no family history of lung disease
 He is homeless and use recreational drugs, takes heroine by injection,
knows about needle sharing programme
 He smokes and drinks
 Have multiple male sexual partners and doesn’t practice safe sex,
never screened for STI and HIV
 Task
 Talk to him and discuss management
Concern
 Did I get HIV?
Examination and management:
61
62

 Take observation ,head to toe


 Examine chest heart and lung and lymph nodes
 (Findings temp high, SPO2 low, bilateral crackles, dull percussion)
 Routine blood , RBS, CRP, sputum for C/S, ABG, chest X-ray , test
for covid
Management
 Greet and confirm the identity
Take focused history
(ODPARA of symptoms)
ask DD (pneumonia, TB, COPD, Atypical pneumonia, acute bronchitis,
pulmonary embolism ) red flags ,
FLAWS ,
associated features leg swelling, leg pain, rash , wheezes etc
Ask MAFTOSA,
DESA
sexual history and drug history,
contact history,
screening
 Do ICE, verbalize examination and summarize,
 explain he has been having a chest infection caused by a bug ,
(probably PCP,to confirm we need to do some test) , this infection is
common in drug abuser and people who don’t practice safe sex and
having multiple partners
 Involve senior
 Treatment: oxygen, as the saturation is low, Broad spectrum
antibiotics(according to hospital protocol)
 Call lung specialist for further management
62
63

 Advice: safe sex and drug abusing , stop taking drug


 Also arrange screening test for HIV, Hepatitis
 Also offer methadone as he has takes heroine

Testicular examination:
History:
 you are FY2 in general practice
 A 32 year old man has come with the complaint of testicular pain
and swelling for last 2 days
 He is suffering the pain in left testis which is swollen, dull aching, not
radiating to other areas and pain score 5/10
 Associated with mild fever and flu like symptoms and swelling in
bilateral parotid gland 5 days back
 No urinary symptoms, no history of trauma
 He is feeling shy thinking of STI , as he had unprotected sexual
intercourse
 vaccination history of MMR is unknown
 no history of FLAWS
 this is the first time he has been experiencing that
 He is otherwise fit and well,
 No discharge, ulcer or swelling on testis
63
64

Task:
Talk to the patient and and take the focus history and do examinations
and discuss management
Examination and investigation:
take observations and verbalize routine blood test
Perform testicular examination,
Do prehn's test, examine the swelling too if patient allows
Examine head neck and lymph nodes (There will be torch but no cone)
Findings:
Examination left side of testicular mass about 1x2 cm in size, firm in
consistency, slight tenderness +
Prehn's test negative
Can get above the swelling
Fluctuation test negative
Trans-illumination test negative
Cremasteric reflex normal
Management:
 greet and confirm identity
Take history
Explore pain,
SOCRATES: on left testis, dull aching, non radiating to other areas and
pain score 5/10
Differentials: trauma, infection, mumps orchitis, STI etc
pastmedical history: flu and fever 5days back
MMA:none
sexual history: not active

64
65

 perform the testicular examination as you were taught


 Do ICE
 explain your findings, as he had flu that was a viral infection cause
mumps caused this pain
 Send home with treatment and advice
 Treatment: offer pain killer
 reassure it’s self limiting, will take week to get over
 Advice:
1. vaccination after he recovers
2. Advice STI screening
3. Using condoms
 Safety netting

Lesion on back:
History:
 You are FY2 in GP
 A 68 year old man has presented with a lesion on his back for 30
years
 Its not painful , no discharge , or bleeding , no itching
 It was pinking red , and becoming dark these days , and he thinks it
changing in color, but no ulceration or skin change

65
66

 recently wife told him to check it,


 he is annoyed by it because he's a swimmer, want to remove it
eventually after telling him it's benign
 No changes , no similar rash anywhere in the body
 FLAWS negative, no medical history
 Family history of cancer
 DESA normal
 he loves sun bathing
Task;
Talk to him and discuss management
Concern:
Is it skin cancer?
Examination and investigation:
Take observation and head to toe , look for rash
Routine blood test, inflammatory marker
Management:
 Greet and confirm identity
Take focused history of rash
size site , shape , surface, discharge , bleeding , painful , itching, color
any changes, ulcer , nearby skin condition etc
Ask trauma , injury,
red flags,
sun bathing, use of drugs , lotion etc
Ask for differentials (SCC, BCC, melanoma, melanocytic nevi, common
warts, acinitic keratosis, seborrheic keratosis)
MAFTOSA ,
DESA,
66
67

do ICE
 Verbalize examination and summarize
 Explain your suspected cause is seborrheic keratosis which is not a
skin cancer, it remains on skin without causing any harm
 Its unlikely to be a cancer , but as he thinks its changing, also family
history of cancer so refer her to dermatologist who will do skin
scrapping to look under microscope to see if anything unusual going
on or not
 Treatment and advice:
1. Give emollient and advice using good sunscreen with PH50
2. Wearing loose fitting cloths covering body
3. Do not scratch or rub
4. If he wants to remove it there are procedure like cryotherapy or
surgical curettage, but NHS doesn’t cover it if for cosmetic purpose
 Safety netting FLAWS and any bleeding or discharge

67
68

Gilbert syndrome
History:
 You are FY2 in general practice
 A 27year old man has come for follow up
 He did some blood test due to yellow discoloration of urine and dark
stool and the result showed increased unconjugated bilirubin and
conjugated bilirubin is normal
 He is not expecting anything serious, had some symptoms of
jaundice and was passing dark stool
 He is generally fit and well , no medical condition or using any
medication
 His father had some liver problem but he cant mention the name
 Eats outside mostly , and drink alcohol within normal limit
Task:
Talk to the patient and explain test result
Concern:
Will my children gonna have it too?
Examination:
Take observation and head to toe
Management:
 Greet and confirm identity and paraphrase the scenario
Ask him what made him to go for the test?
Any new symptoms ?
how is he feeling now?
Explore symptoms
MAFTOSA
68
69

DESA
red flags
 Verbalize examination and summarize
 explain the test result , the condition called Gilbert's , it runs in the
family where the faulty gene means bilirubin is not passed into bile at
the normal rate. Instead, it builds up in the bloodstream, giving the
skin and whites of the eyes a yellowish tinge.
 Assure him its self limiting ,
 advice to drink plenty of water as dehydration is a risk factor
 Refer him to genetic counselling to confirm the diagnosis
 Address concern: Yes, this condition is genetic and it can pass
through your genes to your children
 Safety netting and follow up

Ante-natal check up:


History:
 You are FY2 in obstetrics and gynaecology department
 A 35 year old lady has come for an antenatal checkups
69
70

 she is 36weeks pregnant, and it’s her 4th pregnancy


 Previous pregnancies went well, no complications, delivery were
normal vaginal
 This time pregnancy is going well, no 3rd trimester symptoms, no
bleeding or discharge, no headache,blurry vision,leg
swelling,vomiting,n o past medical history or medication history
 She can feel her babies kicks.
 Mid wife has seen her 6 weeks back and said her baby is in breech
position
 She is quite worried cause she wants normal vaginal delivery
Task:
Talk to the patient,perform relevant examination and discuss
management plan
Concerns:
Can I have normal vaginal delivery?
If the baby does not rotate ?
Do I need surgery?
Examination and investigation:
Take observation, perform antenatal examination as you were taught
Routine tests (FBC, RBS), urine test
Management:
 Greet and confirm identity
 paraphrase the scenario
Take focused relevant history,
previous pregnancies,
recent any newsymptoms,
rule out red flags (headache,blurry vision, fits, ankle edema,
70
71

past medical history and medication history


 Take consent,
 explain examination and perform ante natal examination as you were
taught
 Explain:
1. according to the presentation,if normal she can have normal vaginal
delivery
2. If still in breech we need to wait another week then we will try external
cephalic version twice, if fails then we will go for Caesarean section
 Safety netting

Medical error:
History:
 You are FY2 in AMU
 She was admitted to the hospital for 4 weeks after your colleague saw
her and then was discharged.
 In a previous visit, another GP saw her and diagnosed herwith
pneumonia based on an X-ray. It turned out to be another patient’s x-
ray .The patient’s original x-ray is normal.
 She doesn’t know, you have to tell her about the error.
 She was presented with cough,SOB,and tiredness.

71
72

 No drug side effects, except for some diarrhea that wentaway on its
own.
Task:
Talk to the patient and address her concerns.
Concerns:
I want to file complaint.
Management:
 Greet and confirm patient’s identity.
Take focused history,
hows she doing ,
hows the cough andfever ,
if everything is alright,
any new symptoms
Ask of the side effects and medication
if she has taken all anddeveloped any like rashes , diarrhoea
 Explain the error happened:
1. give pause to express herand go with the medical error structure.
2. Don’t blame anyone in particular, and don’t defend the GP.
3. Try to explain why it happened.
4. Mention that it will never happen again and you will put it on an
Incident form.
5. Connect her with PALS if she wants to complain
 Offer her support.

72
73

ARMD
History:
 You are FY2 in Ophthalmology
 A 79 year old lady was referred by the optometrist who noticed
some age related changes in her vision
 She has mostly gray vision in the centre of the eye , and sees line
wavy while reading books
 No pain , no discharge, no itching , redness , trauma in the eye
 No other associated symptoms like pain in the jaw , headache .fever
flu, etc
 She doesn’t wear glasses , family history of DM , but she doesn't have
any other medication condition ‘
 Eats healthy , doesn't drink or smoke , do exercise
 Read books but having difficulty now, wants to switch in audio books
Task:
Talk to the patient and do management
Concern:
Can I get back my vision?
Examination
Take observation , examine head to toe , neurological examination of
cranial nerve , eye examination , visual field and visual acuity
Management:
 Greet and confirm identity
73
74

Take focused history of blurry vision ,


rule out DD like glaucoma ,trauma, GCA, optic neuritis etc
Rule out red flags,
MAFTOSA,
DESA
 Do ICE
 see the findings and summarize she has been having macular
degeneration probably due to increasing age
 INVOLVE SENIOR
 investigation:Fundoscopy shows drusen macula
 Treatment: depending upon wet ARMD or dry ARMD
 Safety netting for GCA
 Follow up

Viral flu
History:
 You are FY2 in general practice
 A mother of 4 year old boy has called you with some concern ,he
developed high fever , runny nose, for last 2 days
 He was fit and well before that , was going nursery , 4days backhe
suddenly developed fever, she gave calpol but didn’t work , also
cough and lethargy
 He is feeding and peeing well
74
75

 No rash , shyness to light , or other associated symptoms


 BIRDDD normal, no medical condition or medication history, or
allergy, up to date with jabs
 Due of this year flu vaccine
 Mom thinks her son needs antibiotic
Task:
Talk to the mom and address his concern
Concern:
Why you wont give antibiotic to my son?
Management:
 Greet and confirm identity start with telephonic approach
 Take focused history
fever, cough , runny nose: for 2days
redflags: none
differentials , chicken pox, measles. Meningitis , dehydration
BIRDDD: normal findings
MAF: none
 Do ICE
 explain her son has got viral flu which is very common in children ,
and its self limiting
 Send home with treatment and advice
 Investigation: routine blood and inflammatory marker
 Treatment: plenty of fluids and calpol to lower down the fever,
 Advice:
1. can use salt water to clear the nose
2. Keep him feeding good food , and water, check his nappy everyday if
he is passing enough urine
75
76

3. it’s a viral flu , antibiotic wont work, no need of that


4. Tell her to give flu vaccine when he becomes okay
 Safety netting and follow up

76
Flu vaccination
History:
 You are FY2 in GP
 A mother of 1year old child has been calling you for some concerns
 She wants to know about the flu vaccine which her child is in due
 She doesn’t think its necessary as her child is well and active never got
sick
 She doesn’t take the vaccine
 Baby is physically fit , no medical condition
 BIRDDD normal
 She wants to know when flu vaccines are given and side effects
Task:
Talk to her and address her concerns
concern
Why flu vaccines are given?
What are the side effects?
I heard that flu vaccine shot can give you flu is it true?
I have never taken flu vaccine , I never got flu
Management:
 Greet and confirm identity
 Start with telephonic conversation
 Before answering her questions take focused history of the baby
How old is he ?
How is his health in general?
77
Any recent fever flu?
Feeding , wee and poo?
If he is well and active , any other symptoms or complains about his
development
BIRDDD
MAF (head to toe)
 Address concerns:
1. Flu vaccines are very safe vaccine which reduce the risk of flu
illness(influenza and others; and hospitalization among children
2. Reduce the risk of illness and any further complication
3. The yearly flu vaccines are the best way to protect the child from flu
4. It can be given early fall before end of October from the age of 6months
to older (8years); at least two shot in this time frame
5. If someone has low immunity like children , old people , and pregnant
mothers can also take the vaccine
6. It is not true, flu vaccine will not give you flu
7. The side effects of the flu shot can be soreness , headache , mild fever,
nausea ; but it goes away within few days
 Lastly , encourage her to give her child flu shots
 Thank her for calling

78
Dementia
History:
 You are FY2 in in AMU
 A 85 year old lady has been admitted in the hospital because of
weight loss due to dementia for last 4 years, for the last 1 months she is
not taking any food or drink
 Her condition has been deteriorating,
 All er investigations came normal, but she is not improving
 Her daughter has come to see her, she has been taking care of
her mom for these days
 Her mother doesn’t recognize her at all, she doesn’t talk to anyone too,
 Consultant has taken decisions no aggressive treatment at the
moment , only palliative care
 Her daughter is not happy with this decision
 She doesn’t have financial problem , she keeps full time career for her
mother
Task:
Talk to daughter and address her concern
Concern:
How will my mom gonna eat?
Can I take her home?
Examination and Investigations:
All normal including CT scan
Management:
 Greet and confirm identity
 Introduce yourself and build goo rapport
Ask how much she knows about her
79
mother
showempathy
any challenges she has been facing and offer support
Take focused past history
ask dementia questions
treatment, her recent condition
 Explain palliative care, how she can be treated now
 Explain that for her only palliative care is suitable
 Address her concern:
1. if can not eat by mouth, we can provide nutritional fluid
2. Don’t say she is gonna die , but her condition is notimproving, no
aggressive treatment can’t be given
3. If she want to take her mother home , tell her she can but you need
to discuss with the senior regarding that
 offer support

80
81

Prescription
History:
 you are FY2 in in pediatric department
 A 6 year old child is diagnosed with peritonsillar abscess
 Her weight is 22 kg
 Allergic history itchiness in clarithromycin
 No other regular medication to prescribe
Task
Prescribe phenoxymethylpenicillin , metronidazole , Paracetamol
and calculate amount of fluid and prescribe in the fluid chart
Management:
 greet the examiner
 Setting pen , prescription, calculator, BNF, two different stickers
for different form of prescription (fluid sheet andtreatment sheet )
 Prescribe medication from BNF, check BNF for the does
 As no IV phenoxymethylpenicillin , change it to benzyl-
penicillin IV and metronidazole IV
 prescribe paracetamol in as required
 Calculate fluid and fill up the fluid chart ( as you w e r e
taught in the academy)
82

Domestic violence
History:
 You are FY2 in emergency department
 A 22year old male has came to the hospital as he was injured
on his arm
 While asking he said he fell down, nurse has examined him and
found several bruises on his body and he was looking down ,
was not sharing eye contact
 From the notes nurse said he visited previously several times for
injury
 When he was offered confidentiality he said he was abused by
his partner
 His partner is male and has been living together for a year now,
he is 29
 He is dependant on the partner and don’t want to leave him
 Partner is not involve to any crime or drugs
 Didn’t tell to family, don’t want to involve police
 He doesn’t drink or smoke
Task:
Talk to him , assess him and discuss management
Concern:
Dr I don’t wanna leave him , I’m dependant on him
Should I involve police?
Examination:
83

Take observation , head to toe


Check for injury and bruises
Management:
 Greet and confirm the identity
Take history
How did he fall?
When this happened?
How is he doing now?
Acknowledge non verbal ques: you seems down, what happened ,
offer confidentiality
Ask about the partner, age, relationship, how long he is living in
Is he dependant on him? What does he do, who else live
Did he talk to the family
Does he abuse him everyday
sexual abuse
Does he involve in crime , drugs, alcohol
Ask about his mental health , any self harming thoughts
 Explain what he has been going through is can not be tolerated,
domestic abuse is a crime in UK
 Involve social service
 Address concern: Right now we are involving social service;
they will look upon the matter not the police
 Advice
1. to keep his important documents , passports, money in a safe
place
84

2. He can contact GALOP, which provides support where LGBT


individuals experiencing domestic abuse
3. Also there are national domestic abuse helpline, Men’s advice
line to call for help
4. Advice him to talk with his partner, or can come for a discussion
5. Offer him CBT if he seems down , mood low

Change the notes:


History:
 You are FY2 in A&E
 A 30 years old woman came to receive her test result
 She slipped on the grass at her backyard yesterday and was
seen in the A&E,
 She fell on an outstretched hand and had some pain in her hand
 She had bruises, swelling and pain.
 Pain was managed and she was sent home.
 Now she came back to the A&E feeling better ,X-ray was done
showing no fractures.
 She works in a factory , she is mother of two child
 While taking history she said she fell at her workplace, She asks
you to change the note to say that she fell at work.
85

 She tells you that I will get compensation for that.


 Her mood is fine; she doesn't have any medical conditions and
she's totally fit.
 She said she has some financial problem, it’s very difficult for her
to maintain the family
Task:
 Talk to her and address her concerns.
Investigations:
Take observation , examine the wrist
Routine blood test normal
Wrist x-ray is normal.
Picture of xray given
Concerns:
Could you please change the note, I’ll get benefits.
Management:
 Greet and confirm identity.
 Paraphrase the scenario,
Take a focused history of what what brought her to the hospital
hows her hand ,
any new symptoms
Pain
swelling
restricted movements
Check her psycho social,
MAFTOSA
86

DESA
 Explain explain the x-ray, there are no fracture , she got some
pain , take painkillers for that
 When she says to change the note tell her that you can't change
notes, that is against good medical practice
 refer her to the citizen advice bureau, they might offer her
some benefits.
 Advice regarding injury management
1. do not put much pressure
2. avoid carrying heavy object
 Safety net.

lithium toxicity
History:
 You are FY2 in emergency department
 A 63 year old man was brought to the hospital by his daughter
as he was confused since morning
 Also peeing a lot and seems tired
 Daughter is anxious, when you asked he is diagnosed with
bipolar mood disorder for which he has been taking lithium for
6months
87

 He doesn’t have other co morbidity, like DM , HITN , Thyroid


problem
 Retired and Lives with daughter , daughter takes care for him
 No smoking or drinking habit
 No history of dementia, fever, infection, stroke, trauma or fall
Task:
Talk to the daughter , assess him and do management
Concern:
What happened to my father?
Examination:
Take observation, head to toe,
Neurological examination, MMSE
Management:
 Greet and confirm identity
Take history
Access the confusion state
Since when?
what do you mean by confused ?
Exclude dementia by asking: has he been forgetful recently?
Does he struggle to find words ?
Getting lost in familiar places ?
DDs for confusion :
1) UTI : ask about urinary symptoms : going yo the loo more often ?
Any pain passing urine ? Any fever ?
2) Diabetes : Drinking
88

more than usual lately ?


3) Stroke : Any weakness anywhere in his body ?
4) Head injury :
Any recent falls ?
MMA: when lithium is mentioned ask about
Why prescribed ?
Since when ?
Taking as prescribed ?
Going Follow up appointments?
Dose change recently ?
Other side effects of lithium toxicity : Nausea & vomiting? Heart
racing ? Tremors ?
Any other medications ? (to exclude interaction)
psycho social of father and daughter:
does he live alone ?
Any one caring for him other than yourself ?
Is he independent with daily activities?
Who else do you care for ?
Do you find it difficult to care for him?
Any help we can offer ?
 Do ICE
 Explain that you suspect that his confusion is due to lithium
toxicity ,
89

that’s when the lithium level in the blood increases over the
recommended dose ( mention cause if said by daughter , if not then
say we need to do further investigations to see why this happened )
 Admit the patient right away and stop lithium
 Involve senior
 Investigation: FBC, RFT, TFT, U&E, Lithium blood level , ECG
 Management
4. IV fluids
5. Recheck lithium level every 6-12 hours
6. Might need heamodialysis if level is too high
 Refer to psychiatrist for bipolar reassessment and dose
adjustment.
 Advice
4. continuous monitor / follow up appointments
5. no OTC NSAIDs and to maintain hydration by fluid intake
6. Advice not to stop lithium suddenly and always seek medical
advice first .
 Offer support and carer if needed
 Safety net about deterioration , neurological manifestations
( other signs of toxicity)
90

alcohol dependency
History:
 You are FY2 in well-woman clinic
 A 56 year old lady has come for routine check ups
 She is fit and fine, but the mood is low, 6/10
 She s drinking a alcohol a lot, 2bottles of wine and 3/4 glasses
of whisky every week
 she had gone through a separation for 2months now
 She doesn’t want to quit drinking, doesn’t think it’s a problem ,
she feels good when drinks
 She doesn’t smoke or do drugs, financially well and sound
 Lives alone, has one daughter lives in another city
Task:
Talk to the patient and counsel her
Management:
Greet and confirm identity
take focused history
CAGE TWD
 Verbalize examination
 explain she’s drinking more than the maximum recommended
units, which will negatively affect her well being. Walk her through
the side effects on drinking; heart, kidney, liver, pancreas, brain,
socially and mentally.
 Investigation: Do routine blood tests and CBC, KFT, AST, ALT,
GGT, and Amylase to check for any negative impact.
91

 Offer talking therapy (CBT) and alcohol anonymous support


group.
 Offer family and friends support
 Involve senior before prescribing medicine
 Treatment:
1. Chlordiazepoxide (withdrawal symptoms)
2. Acamprosate (cravings),
3. Disulfiram (deterrent).
 Advice:
1. Tell her what to expect during withdrawal period and how to
cope. The symptoms are worst for the first 48 hours and last for
3 – 7 days from the last drink. For example feeling stressed and
sleep disturbance.
2. Drink plenty of fluids (3 liters a day), eat fruits, eat regular meals,
and avoid caffeinated beverages.
3. Inform the DVLA and don’t drive during this period.
4. Keep alcohol diary.
 Safety net for low mood and cravings
92

Smoking cessation
History:
 You are FY2 in OBG and gyne
 A 32 year old mother has delivered a baby boy 1week back
 It was her first child , baby and mother is doing well
 Baby born full term weight with healthy weight, no jaundice , infection or
any abnormality or complication during birth,
 Mom is also fine , no medical condition or medication she has been
taking except supplements
 Baby is well breast feeded
 Mom is a smoker since her teenage , has been smoking 20 to 30
cigarettes per day
 She doesn’t do drugs , or drink, her husband also smoker, they live in
the same house and taking care of the baby
 Mother couldn’t stop smoking even after the child's birth
Task:
Talk to the mother and counsel her to stop smoking
92
93

Concern:
How smoking can harm my baby?
Can nicotine pass through breast milk?
What if I smoke e-cigarettes; will it be harmful too?
Management:
 Greet and confirm the identity, start with telephonic approach
 build good rapport, congratulate her for being new mom
Take focused history
ask about baby's well being
if he is okay
any issues with wee and poo
if he is feeding well
any fever or other symptoms
sign of infection anywhere
gently ask the mother if she is smoking , or reduced
tell her why she needs to stop smoking for her and babies well being
Ask about her partner too
he needs to stop too
 Explain passive smoking or Secondhand smoke can cause sudden
infant death syndrome (SIDS), respiratory infections, ear infections, and
asthma attacks in infants and children.
 Convince and counsel her to quit smoking from today
 Explain different method like nicotine patch , gum, cigarettes etc
 Also she can join different social group to have good start and help
 Adress concern:

93
94

1. Smoking harm newborn babies as they breathe in harmful chemicals


from the smoke causing breathing problems
2. Yes , nicotine can pass through breast milk when mother smokes, it can
potentially harm the baby’s health and development
3. E-cigarettes can still harm newborn babies as they contain harmful
chemicals like nicotine.
 Tell her she can contact anytime for any help

Simman
History:
 You are FY2 in emergency department
 A 62 year old man has been brought to the hospital as he hasbeen
feeling unwell, have burning sensation while peeing and high fever for
2days
 He is confused since morning
 There is no cough, shortness of breath or chest pain
 Patient has a history of prostate surgery 1week back, urinarycatheter
and bag was attached since then
 No known allergies
 Talk to examiner at 6 minutes bell
Examination and Investigations:
 Perform ABCDE approach
 Temp high, BP 100/70, spo2 96%
94
95

 Routine blood test, LFT KFT, urine R/E , dipstick, culture,inflammatory


marker, lactate, 24hour urine output
Management:
 Greet and confirm identity, take universal precautions
 Take focused history ( what happened, happened before, medical
history ,medications, allergies)
 As patient is drowsy Perform ABCDE
 Take 3 and give 3 of sepsis (serum lactate, urine output,urine
culture, Antibiotics, Fluids Hartman solution 1L in 15 minutes and
1L in 45 minutes)
 Admission and senior consultation for further assessment and
management accordingly

Psychosis
History
 You are FY2 in GP
 A father of 30 year old son has booked an appointment for him
 He has been behaving abnormal for past few days
 He has become very talkative, and saying unusual stuff
 It’s happening since he lost his job 95
96

 Patient talking by himself; he doesn’t think he has any problem


 At first he did not open up, then showed some non verbal ques that is
looking outside
 He mentioned a car is following him for few days and there is a man
inside , he never saw his face; he has a camera and recording him and
following him
 He doesn’t know why he is following him, or spying on him
 He is not on any drugs or alcohol
 General health fine , he is fit, no recent infection
 He refuses treatment , he thinks he doesn’t need any medical
assistance, he is fine at home
 He is not carrying any weapon
 No aggression towards anyone, no suicidal thoughts
Task
Talk to him and discuss management
Concern
I don’t think I have a problem, I don’t need any medical treatment
Management :
 Greet and confirm identity, paraphrase the scenario
Take history (follow the non verbal ques first)
(since when he has been seeing these , do you know why he is following
you, did he see the persons face, did he tell him anything or contact with
him anyway, anything else )
 ask MCFAMISH questions
 Start with cognition and hallucination questions first

96
97

(mood , suicidal thoughts, hallucinations, aggression, impact , insight,


drugs , alcohol, support etc), then rest
Explain what you have found
Now explain him, he has been suffering from some mental health issues
(Acute psychosis can be described as a clinical syndrome that involves hallucinations,
delusions, disorganized thoughts or behaviors, or some combination of these within
an acute time frame , often less than 1 month ) which needs immediate medical
help;
1.admission needed (As he is in GP, tell him to come to hospital, if he
refuses to come to hospital then apply refusal approach and send
ambulance to bring him hospital )
(Tell him we want to help you, you will be safe, we will find what’s going
wrong and will treat you accordingly, you might need some medication and
behavioral therapy)
2.Investigation (after admission all blood, urine dipstick, test for drug and
infection)
3.Involve senior
4.psychotherapy and CBT needed
5.Anti psychotic (these will be given all after admission in hospital, not in
GP)

97
98

HIV
History:
 you are FY2 in GUM clinic
 35 years man presented with lymphadenopathy and have tests done.
HIV positive, chlamydia negative.
 he travelled to Thailand a month back with one of his friends. Henoticed
swelling in his inguinal region, armpit and also had flu like symptoms 2
weeks ago.
 Also complained of tiredness and not feeling well
 He had sex with a prostitute, didn’t take protection
 He is married for 3 years but have no children. He also had unprotected
sex with wife.
 no past medical condition, medication history
Task:
take history, address his concerns, discuss management
Concern:
Should I inform my wife?
Is there any chance that I could have transmitted HIV to my wife?
Examination and investigation:
Take observations, head to toe (swollen groin area,Generalized
lymphadenopathy)
Management:
 Greet and confirm identity
 Paraphrase scenario, take history of last visit, explore symptoms, ask
about recent symptoms
History taking:
Generalized lymph nodes in inguinal region
98 , armpits
99

Fever for 2 weeks


Tiredness and malaise for same duration
No associated symptoms
MMA: N/A
sexual history: had unprotected sexual intercourse with prostitue
Married , had unprotected sex with wife recently
 Verbalize examination and explain test results:
he is HIV positive, (start with BBN approach) He needs to inform your wife
because there is a risk she might have acquired it, also if she already has it
then it would be better for her to start treatment as early as possible
 Send home with advice and treatment
 Investigation:
1. blood test to check the viral load of HIV (HIV viral load test)
2. CD4 lymphocyte cell count
 Treatment: antiretroviral medication
 Adress concern:
1. mention about the contact tracing of your partner inThailand and inform
your wife through partner notification program.
2. There is chance of passing HIV through unprotected intercourse
3. There is treatment for HIV infection but no cure. The medication will
weaken the virus and reduce its effect on the immune system.
 Advice
1. regular check up of blood, level of infection which is known as a viral
load
2. Anti viral medication to keep the viral load low. If he doesn’t take the
medications, then he can have serious complications.
99
10
0

 Safety netting
 Follow up in a week

Non accidental injury:


History:
 You are FY2 in pediatric department
 A 30year old lady has brought her 3months old child in the hospital with
swelled arm, she noticed that in the morning while changing her cloths
 She lives with her boyfriend , he is not the biological father
 everything is normal in the pediatric history of the child .
 her boyfriend said that the child had a rough night. You brought thechild
immediately.
 Her partner works in a bar. He doesn't have financial problems, She is
not in a contact with her biological dad.
 Baby is doing fine, no medical problem , or medication history
 This is the first time happened
 An xray done shows spiral fracture of the arm
Task :
Talk to the mom and discuss management
Concern:
How this is happened?
Investigation:
Routine blood, xray, skeletal survey
10
0
10
1

Management:
 Greet and confirm identity
Take focused history,
when did you notice the swelling?
How did you notice it?
Do you have any idea how he could have sustained injury?
Is there any chance that he fallen down?
What did you do immediately?
What time did you bring him/her to the hospital?)
Explore home environment,
relation ship of her partner and baby,
BIRDDD,
MAF
 Do ICE,
 explain we have done xray and shows fracture we call it spiral fracture ,
which is a bit unusual in this case cause it needsa big pressure on bone
 Involve senior,
 ask for more investigation to rule out NAI
 Tell her that we want to involve social service to check if everythingis fine
 For the baby she will be admitted and will be under observation
 She doesn’t need any surgery, as it will heal by its own

10
1
10
2

Teaching
History:
 you are FY2 in in emergency department
 A 5th year medical student who missed the BLS workshop
 He has no idea about it , now he wants to learn
 ask him to perform and give him a feedback about it.
 Scenario says there is a patient who has collapsed in the hospital
corridor, what will your first step to save the patient
 Set up: adult CPR mannikin, face shield.
Task:
Teach the student how to perform basic life support
Management:
 Greet and build good rapport
 paraphrase scenario and appreciate as he has come tolearn.
 Ask about knowledge and understanding, explain BLS ,when we do it
 A patient who has collapsed in the hospital corridor, your first step to
save the patient will be removal of danger stuff around the victim
/patient
 Check for patient’s response
 Shout for help , asses airway, check breathing , circulation, chest
examination
 Start chest compression, perform it as you were taught
 After 30 chest c o m p r e s s i o n give to mouth breathing after
pinching the nose
 Continue the process and reassess
 Call ambulance 999
10
2
10
3

 Indication of stopping if ambulance arrives, if patient shows sign of life,


if you are tired
 Seek for help, ask for student’s understanding
 Tell him to read more from resuscitation UK

Breast lump
History :
 You are FY2 in general practice
 A 42 year old woman has presented with some concerns , she has
noticed a lump on her right upper quadrant of the right breast few days
back
 She is a bit concerned and worried and come for a visit
 Lump is very small , soft , not attached, not increasing in size or
shape, no swelling , changes in the breast , any changes in the nipple
or any discharge, no lumps and bumps anywhere in the body ,
FLAWS negative
 MAFTOSA DESA normal
 No positive family history of cancer or breast cancer
 Her menstruation cycle is fine, LMP was 2weeks back , no use of
contraceptive except condom
Task:
Talk to the patient and examine the relevant
Concern:
10
3
10
4

Do I have cancer, as my friend got breast cancer and she noticed a lump
in her breast
Examination and investigation:
Take observation and head to toe
Perform breast examination as you were taught
Finding there is a small nodular lump in the right breast
Take routine blood
Management:
 Greet and confirm identity
Take focused history of lump
ODPARA
Right site
Noticed suddenly on right breast
Not attached , soft lump, not increasing
related history: no bleeding or discharge from the breast, no fever, no pain
changes in the breast ,nipple: no change
FLAWS : negative
MMA: N/A
P4: LMP 2 weeks back
 Now verbalize examination and PPCCE, explain the procedure and
ask for chaperone (examiner will be the chaperone)
 Now perform breast examination inspection , palpation , inspect the
lump , and explain finding , its not something harmful from the
examination and history
 Involve senior and arrange a mammogram
 Advice for self breast examination
10
4
10
5

 Safety netting FLAWS

headache
History:
 You are FY2 in general practice
 17 year old lady has come with headache which is dull,continuous
 headache usually starts before her period and stopsabout 2 days
into period

 she tried ibuprofen and paracetamol but it didn’t help


 No other symptoms
 menstruation regular, otherwise fit and well
 no smoking or alcohol habit
 Diet normal
Task:
Talk to her, address her concerns and discussmanagement
Concern:
Dr, I am worried if I take steroids they would increase my weight
Examination and investigation:
Observations, Fundoscopy, Nerves of head and arms ,Ear and nose
Routine bloods ,ESR and CRP
Management:
 Greet and confirm identity
10
5
10
6

Explore the headache,


ODPARA
Do differentials (migraine, SHA, meningitis, tension headache, GCA,
cluster headache, sinusitis etc)

ask pre menstrual symptoms questions (breast tenderness, loss of


appetite, mood swing, trouble sleeping, low sex drive)

ask menstrual history, sexual history, pills, mood

Do idea, concerns;

verbalize examination and investigation

 Explain menstrual migraine which precipitated by period

 Treatment: Offer sumatriptan during the attack of migraine as ibuprofen


and paracetamol are not working

 Once the relationship between the headache and menstrual period has
been confirmed then she needs to start on COCP to be taken
continuously.

 Advice:

1. Advice of keeping a diary of period and headache

2. yoga and exercise

 Refer to dietitian for diet plan

 Follow up in three months

 Safety netting

10
6
Blood test result:
History:
 you are FY2 in general practice
 A 40 year old woman has come for well woman check up a week
ago and has done some blood test, whichshows low MCV and low
HB, others are normal
 she has been feeling tired for few months
 Physically fit and well, no other symptoms
 No smoking or drinking habit
 Her sister has thalassemia
Task:
Assess patient and discuss the initial management withthe patient.
Examination and Investigations:
Take observations and head to toe, heart , tummy
Management:
 Greet and confirm identity
 Paraphrase the scenario ,
explore tiredness
recent conditions
any new symptoms
exclude red flags
MAFTOSA
DESA 107
period
previous surgery
 do ICE,
 Verbalize examination and summarize
 explain test results which is normal except for Hb levelis low and the
volume of your blood cells is also low which indicate that she might
be having thalassemia & this is what causing her tiredness (positive
Family history, if she was only a carrier, she won’t have symptoms)
 Refer to blood specialist for reassessment
 further investigations :genetic testing, Hb electrophoresis, skeletal
surveys like x-rays and scans and also for further management
 Avoid Iron supplements except prescribed ones after tests.
 Encourage family to get genetic counseling and testing if her
condition becomes confirmed.
 Safety netting: dizziness, fatigability, bleeding, bruising

Anal fissure

History:
 You are FY2 in GP
 A 30year old lady who is 23 weeks pregnant came with the complain of
per rectal bleeding and pain while passing stool
108
 Its been for a week, she is been little bit constipated.
 Pain is not that severe, doesn’t go anywhere, now getting worse
 This is her 2nd pregnancy ; it had in her 3rd pregnancy as well
 Blood is fresh , streak blood with stool
 Patient was sitting comfortably, fit and well, no fever weight lose
 She said she is using lidocaine jelly by herself
 No medical condition , taking only supplements
 Diet fine, no smoking , no drinking
 Pregnancy going well, up to date with check ups
Task:
Talk to her and discuss management
Concern:
How can you help?
Examination:
Take observation , head to toe
Per rectal examination(findings given, visible fissure)
Management:
 Greet and confirm identity
Take history
Bleeding per rectum
Since when?: for a week
How much: copious
Color of blood; bright red
Amount: scanty, streak of blood
Frequency: for a week, several times
Type of stool/ bowel habit: constipation
Associated symptoms: pain while passing stool
Ask about the toilet habits 109
Ask DD (trauma, haemorrhoids, constipation, perianal abscess, TB etc)
MMA: none
Pregnancy: 2nd Gravida, going well, no pregnancy complications
DESA: fine
 Do ICE
 Verbalize examination and summarize
 Explain you are suspecting anal fissure which is a tear or open sore that
develops in or around anus; risk factors are anal sex, constipation ,
pregnancy, STI, persistent diarrhoea etc
 Send home with advice
 Involve senior to rule out haemorrhoids
 Treatment: lactulose for hard stool, pain killers, she can use lidocaine
gel
 Advice:
1. Assure her most of the anal fissure becomes alright by its self
2. Increase daily intake of fibers
3. Avoiding dehydration by drinking of plenty of water
4. Do exercise , yoga, and you can do it in pregnancy
5. Do not delay going to loo if feel for urge
6. Avoid product contain alcohol to wipe; and avoid wiping to hard
7. Soak the bottom with warm bath, towel, keep it clean and dry
 Follow up after few weeks
 Antenatal check up as well

110
counselling

History:
 you are FY2 in in general practice
 a 52 year old has come to you with some concerns , shewants to lose
her weight
 She has always been fat since childhood and all theother family
members are fat too
 she always feel hungry and eats a lot mostly fast food
 She works as a secretory and doesn’t get time for gym
 she is a single mom and have 2 children.
 her BMI is 40 and weight is 100kg.
 She used to go to the gym but have stopped becauseshe feels
embarrassed as people laugh
 her period has stopped at the age of 35 as the ovaries failed early.
 She doesn’t have any other medical condition, she took medication that
says to make her slim but didn’t work atall
 She doesn’t smoke or drink
 Her mood is fine 6/10, but she is struggling to lose weight that’s her
main concern
 she never went to the dietitian, she said her work life too busy she can’t
follow any diet chart

111
11
2
Task:
Take a history from the patient and address her concerns
Concern:
I want to lose my weights
Any surgical option available?
Examination and Investigations:
Take Observations , head to toe, Chest heart , thyroid BMI
Routine bloods (RBS, LFT, TFT, KFT), hormonal level lipid
profile, ECG, Fundoscopy, urine dipstick
Management:
 greet and confirm identity
Explore presenting complaints,
since when
psycho social, mood
what she has done so far for losing weight
ask lifestyle, diet , daily life , family, support , anystress in life etc
Ask about health condition
any Co morbidity hormonal problems etc
MAFTOSA DESA
ask concern and expectations
 verbalize examination and summarize that she has familial
obesity, which run in families.
 Advice: Diet, Exercises
 Refer to gym instructor to provoke exercise training

11
2
11
 refer to Dietitian, she will make a suitable diet plan for her
3

 Medications: can be started on Orlistat to prevent absorption


of dietary fats in foods, side effect of orlistat: fatty stool,
abdominal discomfort, bulky stool.
 option for surgery: like a band is tied around the stomach to
reduce the amount of food that enter and also another
option is to remove a part of the stomachto make it smaller.
 Refer to a psychiatrist to help her mentally
 Follow up for hypertension and atherosclerosis
 Safety netting: Low mood, generally feeling unwell
 Join a local weight loss group: she can do activities like
walking and jogging

Learning disability
History:
 you are FY2 in MOPD
 a 35 year old man has come for a review,he w a s d i a g n o s e d
w i t h t y p e 1 diabetes 3 weeks back
 He was prescribed 2 types of insulin.
 Short acting insulin 3 times withmeals and long acting insulin.
 He sometimes skip meals because he doesn’t like eating.

11
3
11
 Nurses come to his house but sometimes he is out of house so he
4

missed the appointments


 He is currently unemployed,but managing financially
 He is non smoker and a social drinker
 He has learning difficulty.
 He is slow in understanding the information and you keep asking the same
Q to the Drs because he is finding it difficult to remember things.
 He only takes insulin when your glucose is high.
 He is happy if the Dr says he will write down everything for him.
Task:
Asses the patient’s understanding about his condition and discuss
management plans
Examination and Investigations:
Take observations and head to toeExamine chest,eyes, legs
Routine blood test, LFT, KFT, RBSBlood sugar is high
Management:
 greet and confirm identity
Paraphrase the scenario,
take previous focused history,
how is hecoping,
how is he doing,any more symptoms,
any challenges,
Take medication history,
does he check his glucose regularly
ask complications (eyes,urine,leg,heart,foot),
ask pasthistory,
MAFTOSA,
11
4
11
5
DESA,
driving
 do ICE,
 verbalize examination and summarize
 his blood sugar is onthe higher side cause he is not taking the insulin as it
is prescribed
 don’t use any medical jargon s,be slow in explaining and check
understanding
 Explain the medication, how to take, how it works, taking meal after
insulin and measuring blood sugar
 refer to the dietitian
 Advice
1. lifestyle modification,
2. consider wearing a medical emergencyidentification bracelet or
similar
3. advice of the need to contact the DVLA to inform them o f t h e diagnosi
4. Advice patient to carry insulin in their hand luggage i f t h e y are
traveling
5. he will be reviewed by the diabetic nurse to explain the medication.
 Safety netting

11
5
116

lung cancer

History:
 You are FY2 in acute medical unit
 A 64 year old man has been admitted in the hospital after acute
exacerbation of COPD
 He has been diagnosed COPD for 10years
 He is not feeling well for last few months, has symptoms like
coughing , respiratory problem , difficulty in breathing, chest
discomfort which worsen at night
 His cough is productive , copious in amount and he found fresh
blood in it on 3occassions
 He has lost some pounds in last few months, lost his appetite, also
find difficulty in swallowing
 He was a chain smoker for 40years, and drinks alcohol
occasionally, he has lowered the number of cigarettes but couldn’t
stop it
 He doesn’t have any other disease , or allergy , family history of
cancer positive
 He is a retired officer
 He has admitted in the hospital previously many times due to chest
infection and respiratory problems
Task:
Take history, address concerns and discuss management
Examination and investigation:
116
117

Take observation , head to toe, chest examination , examine the


lymph nodes
Routine blood, Inflammatory marker, ABG,
chest xray, ( a hilar mass, nodule in the lung, mediastinal
lymphadenopathy),
clotting profile, RBS
Low sodium in blood
Management:
 Greet and confirm diagnosis
Paraphrase the scenario
how is he doing now,
any new symptoms,
what brought him to the hospital
Take focused history of the symptoms,
ask about cough ,
amount of blood , color, associated symptoms etc
ask FLAWS,
MAFTOSA,
DESA
 Do ICE, verbalize examination and summarize ,
 explain the information he has given and the chest xray relates to
something sinister, it can be a lung cancer, give a pause, let him
think and answer to your question
 Involve senior
 Call specialist to review and perform invasive procedure like
broncoscopy and biopsy, also a CT scan
117
118

 If it is confirmed , we will go for either surgery then chemotherapy


and radiotherapy
 Offer support, and ask any family member he wants to talk

118
119

Insomnia
History:
 You are FY2 in GP
 A 33year old man has come with sleeping problem , its been for 3
months now
 It started when his wife left him , as he cheated on his wife
 He is living in extreme guilt and can not sleep at night
 He sleeps late night , and wakes up early, doesn’t feel fresh, sleeps
around 3 to 4hours a day
 He feels sleepy at day time but he can not sleep due to work
 He doesn’t feel like working anymore, or doesn’t find any energy to
do anything
 He is living alone, didn’t discuss anything with anyone
 He also started drinking much alcohol , more than 14units a
week
 He also smokes cigarettes, not using drugs
 His mood is low, 5/10, but doesn’t have any suicidal thoughts
 He feels hopeless for his guilt, always coming his wife’s thoughts
 No health issues, MAFTOSA normal
Task:
Talk to him and discuss management
Examination and investigation:
Take observation and head to toe
Routine blood test
Management:
 Greet and confirm identity
119
120

Take focused history of insomnia,


since when ?
is it everyday ?
what happened before that,?
any incident ,?
any trauma, ?
when you fall asleep,?
when you wake up, ?
do you take nap at day time ,?
how do you feel in the morning,?
do you drink before going to bed? ,
what do you do usually to fall asleep?
, ask about sleep environment etc
ask about mood,
rule out suicide , hallucination
Psycho social and support system ,
hows this effecting on his life?
Can he concentrate on work , ?
how about his interest at work?
2major symptoms and 4minor symptoms of depression
MAFTOSA, DESA,
any drug abuse
 Do ICE,
 verbalize examination and summarize , explain he has been
experiencing depression induced insomnia
 Offer counselling session (CBT) to cope up with his condition
120
121

 Advice
1. share to friends and families to have support
2. Take a break from work and give himself more time , go on a
vacation, stop thinking much about the incident, divert his
concentration to something he likes to do , like his hobbies
3. Stop drinking alcohol, drinking will worsen the situation , it will make
him more upset now, and also cut down smoking
 Treatment: Prescribe him some anti depressant medication and
advice not to stop taking it without doctor’s advice
 Tell him for follow up after 2weeks
 Offer crisis card , if anytime he feels to talk or need help he can
call on this number on the card

121
Per rectal examination:
history:
 You are FY2 in general practice
 50 years old man came to well man clinic for follow up
 He had frequency of micturition during the day for 6 months
 No other symptoms
 Recently a friend got prostate cancer that’s why he came for check up
 Otherwise fit and well
 He has diagnosed generalized anxiety disorder and under
treatment
 FLAWS negative, no fever, no other urinary symptoms, no past medical
condition (DM , RENAL STONES, BPH)
 No family history of cancer
 He wants to do the PSA test , he is worried he might have the cancer
Task:
take focused history, perform examination and do the management
Concern:
He denies the DRE, asks why its necessary to perform the rectal
exam?
Examination and Investigation:
take Observations , do head to toe
Perform prostate exam and abdomen
Management:
 Greet and confirm the identity
take focused history 122
u r i n a r y symptom: frequency
FLAWS: negative
MMA: none
 explain examination, take consent, offer chaperone
 When the patient denies gently explain why its necessary as he has
urinary symptoms; so examining him before ordering PSA is to be done
to overcome the false positive result
 Perform the Per rectal examination as you were taught
 thank the patient and summarize what you have found in the mannikin
 Investigation: routine blood , including PSA
 Explain PSA test is not reliable for prostate cancer diagnosis
 Follow up in 2weeks
 Follow up with psychiatrist (as patient was very anxious thinking
he has cancer)
 Safety netting FLAWS

123
Prescription:
History:
 you are FY2 in hospice care home
 A 78 years old female is diagnosed with metastatic
Colon cancer
 patient is terminal, palliative care has been prescribed,
she has been referred from hospital to hospice for the
continuation of palliative care. She can not eat or drink
 Her list of medications can be found in the hospital
 Handover will be inside the cubicle
Setting:
 no patient, only examiner , start writing prescription,
BNF , Pen, calculator
 Morphine for pain via syringe driver 30mg per 24 hours
subcutaneous injection
 Cyclizine 50mg TDS SC for nausea and vomiting
 Midazolam 2.5 mg SC 2hourly for agitation
(maximum doses 6)
 Hyoscine butyl bromide 400 micrograms SC 2-4 hourly
for secretion
 Paracetamol 1gm PO every 4 to 6hourly for pain
 Atorvastatine 10mg PO once daily
 Prescribe morphine breakthrough doses

124
 All as required
 Patient under palliative care, don’t prescribe the oral
medication, only subcutaneous injections

Asthma
History:
 you are FY2 in emergency department
 A father of 10 years of son has been calling you
 His son has been having breathing problem since
morning
 There is no fever , no rash , no shyness to light , no
sign of dehydration
 He was fit and well yesterday, wee and poo fine
 No changes in color of lips or skin
 He has been diagnosed with asthma since childhood ,
takes inhalers
 Father has given inhaler , didn’t work
 Saturation was not measured
 He is alone at home with his boy
Task :
Talk to the father and do immediate management
Concern:

125
What is happening?
Management:
 Greet and confirm identity, start with telephonic
approach
Take focused history,
ODPARA of presenting complaint
ask red flags, meningitis , encephalitis , dehydration, fever
flu etc,
ask MAFTOSA,
Any injury,
past history
has it happened before?,
any allergies
 call an ambulance and assure him ,
1. tell him to relax his child,
2. keep him in propped up position ,
3. loose the cloths ,
4. open windows ,
5. nebulize him if possible , measure saturation if
possible
 Explain it is acute exacerbation of asthma , he needs
hospital admission , ambulance has been called will be
arrived within 5minutesat his home

126
 Treatment: He will be given oxygen, and salbutamol,
IV channel will be made togive him IV medicine and
fluid , also he will be monitored
 Assure him and appreciate for calling

BBN
History:
 You are FY2 in GP
 A young male presented with bone pain and swelling in
his lower limb for over a month and presented 2weeks
back to the doctor
 The pain started gradually and increasing , doesn’t go
anywhere, nothing makes it better, exertion makes it
worse , he tried ibuprofen doesn’t help
 He is normally fit and well no other medical condition ,
no associated symptoms
 He doesn’t have any idea why this happened, he is
having difficulty in walking
 Blood tests and X-ray of bones has done
 No positive finding in MAFTOSA DESA FLAWS
 written test result says lytic lesions with sun burst
appearance suggestive of osteosarcoma

127
Task:
Talk to him and discuss management
Concern:
Whats happening? After hearing of cancer, he was
shocked and worried
When I will be seen?
Is it curable?
Examination :
Take observation and head to toe
Management:
 Greet and confirm identity
 Paraphrase the scenario
Take history
ask present condition : gradually started pain , progressing,
more on exertion, doesn’t go anywhere
pain , swelling , any improvement:
MMA: N/A
bone disease: no
 Do ICE
 start with breaking bad news approach
 explain the xray result and what is osteosarcoma, it is a
type of bone cancer ; Give a pause ask his concern
and answer them , the cause of it is unknown , mostly
changes in DNA, or underlying genetic disease etc
 48hours urgent referral to the specialist

128
 Investigation2: PET-scan, biopsy
 Treatment : chemotherapy- surgery- radiotherapy
depending on prognosis
 Address concerns: Assure him; It improves with the
treatment and needs lifelong monitoring
 Advice:
1. People recovering from bone sarcoma are encouraged
to follow established guidelines for good health, such
as not smoking, limiting alcohol, eating well, exercising
regularly, maintaining a healthy weight, and managing
stress.
 Safety netting for fever , weight loss, exhaustion ,
redness around the joints

129
confusion
History:
 You are FY2 in GP
 A 70 years old male presented with complain that he
feels unwell at times
 It has been going on for a few weeks and he has had 2
similar episodes before
 Last episode was yesterday while he was playing golf
with his friend
 Previously he has been having trouble with his bladder
for which he went to the doctor
 He had been going to the loo more frequently for the
past few weeks and the doctor prescribed him
oxybutynin
 He has been taking double dose of medication for the
past few weeks
 He is feeling fine at the moment
 His urine symptoms have also improved
 He has no other medical condition and is not on any
other medication
 No symptoms of any active infection
 He wanted to know if the medication has something to
do with this
Task:
Talk to the patient and address his concerns

130
Examinations and Investigations:
Take observation , (normal) , tummy examination
Do GCS
Routine blood, urine R/E , CRP, electrolyte
Concerns:
Is it dementia?
Management:
 Greet and confirm identity
Take focused history
(confusion, medical conditions, medications, dementia,
sepsis),
Rule out red flags and risk factors
Explore bladder symptoms
medication use
 Verbalize examination including cognition and
neurological exam
 Explain that the medication he is using for the bladder
problem could be a cause of these spells )
 Explain that it is very less likely to be dementia as it is a
progressive disease and does not show up in such a
short time without any history of such symptoms
 Involve senior
 Stop oxybutynin and give alternative medication
(like tolterodine)
 Safety net

131
UTI 3year old
History:
 You are FY2 in general practice
 A mom of 3 year old kid has come to the hospital with
some concern, her child his peeing a lot these days
without drinking much
 Passes smelly urine , he was a bit feverish for last
3days
 No blood in the urine , bowel fine , feeding well , no
rashes , no shy tolight
 No medical condition or medication history , no allergy ,
up to date withjabs, development well, no significant
birth history
 Baby is a bit fussy , irritates and cries while peeing ,
feels discomfort , he is potty trained
 Not playing that much
 Mother is worried
Task:
Talk to mother and do management
examination and investigation:

132
Take observation and head to toe
Routine blood , RBS, urea electrolytes, Urine dipstickXray
of tummy
Management:
 Greet and confirm identity
Take focused history
since when
exclude red flags
Any previous attach
Exclude symptoms , treatment of previous attacks
Rule out recurrent infection
ask head to toequestions
bowel bladder
Dehydration
BIRDDD
MAF
 Do ICE,
 verbalize examination and investigation ,
 dipstick Shows leukocytes an nitrates
 Explain mom that he is having an infection in the urine ,
which is very common
 assure her , and tell her to give him plenty of fluid to
drink,
 Maintain hygiene, give calpol for fever and tummy pain

133
 Treatment: Offer antibiotic according to the hospital
protocol (usually trimethoprim)
 Follow up
 Safety netting dehydration , vomiting high fever

teaching
History:
 You are FY2 in surgery department
 A 5th year student has come to you to know about
taking consent
 He wants to know how essential is it to take consent
and when to take it
 He wants to know GMC updates in surgery cases
Task :
 Teach the student how to take consent from a patient
and its importance and address his concerns
Concern:
When we will take consent?
What will happen in case of children and old people not
having mental capacity?

134
GMC updates in surgery cases taking consent
Management:
 Greet and introduce yourself
 Build good rapport and ask about hows his study going
, praise him for coming to learn about something very
important to know while practicing medicine
 Explain: what is the meaning of giving consent, a
health professional must take consent from all the
patient before providing any treatment , or performing
any procedure or any surgery (for example giving blood
sample , donating organ etc)
 Someone has the right to refuse the treatment and you
need to respect that, no matter what will be the
outcome of it (even death)
 Take the consent orally and in some cases written
(while performing surgery and organ donation etc)
 While asking for consent the decision from the patient
should come voluntary without any pressure from
doctor, friends or family
 Address his concern:
1. a patient can refuse treatment with having full mental
capacity , but if someone lacking mental capacity (like
mental health condition, dementia , learning disability ,
brain damage, alcohol or on drug etc) in that case if the
patient doesn't have anyone to take decision on behalf

135
; a health professional will think what will be the best for
him or otherwise that patient’s next to kin , or power of
attorney can make decision
2. There is something call advance decisions or living will.
This is a decision to refuse a particular medical
treatment for a time in future when the patient might be
unable to take the decision; patient having more than
18 of the age and having full mental capacity can take
such (like CPR)
3. If he patient take a life sustaining treatment in the future
, the advance decision needs to be written down in a
form and signed by the heath professional and a
witness
 Check his understanding and answer his questions
 Thank him and offer him some some links about
informed consent to have more knowledge

136
celiac disease
History
 You are FY2 in OPD
 A 28 year old woman who was referred by the GP
 she had few months back to back altered
bowelchanges, stools hard to flush.
 Has already symptoms of tiredness. No rash. no
othersymptoms, physically fit and well
 No past medical, medication history, no smoking or
drinking habit, no allergies
 Her sister has celiac disease.
 she was not particular about her diet and eats
everything so she wasn't sure what the trigger was.
 She is confused if she really has celiac disease
because the TTG came positive.
 She will be happy to know about before and after the
procedure. She knows a bit about the endoscopy as
hersister has gone through this procedure.
 she seemed overwhelmed especially about the diet
prior to biopsy
Task:

137
Explain endoscopy to the patient and address her
concern
Concern:
I’m not comfortable talking to doctors
Why do I need to eat gluten food again before the biopsy?
why do we still need to confirm through biopsy?
What to expect during the procedure?
Examination and Investigations
Take observations, head to toe and tummy examination
Management:
 Greet and confirm identity
 Paraphrase the scenario
take focused history
how’sthe symptoms now
DESA,
MAFTOSA
Ask and address concerns and understanding
 Explain the purpose of the endoscopy, the test doesn’t
confirm the diagnosis hence we do a camera test called
endoscopy.
 Explain what Endoscopy is, it is a camera test done to
see the inside of the bowel. This will help us to
understand what may have gone wrong and
sometimes

138
 understand what may have gone wrong and
sometimes take a tissue sample for biopsy.
 Pte-requisite:
1. Explain the pre procedure preparations, taking gluten
containing food twice daily for 6 weeks.
2. It is important you eat food which contains gluten like
bread, cakes, biscuits. Be aware that these types of
food can cause bowel inflammation but if you don’t eat
these foods we will not find the cause of the
inflammation and this would give a false result.
3. while the procedure she will be given sedative to make
her relax, it lasts 20-30 min.
4. Complications is rare , but bleeding ,pain can occur
Adress concerns:
1. You need to eat gluten contain food to get a positive
test result to proof the disease
2. Biopsy is still necessary cause doctors can see the
changes in tissue of guts to confirm the diagnosis
 Take consent , safety netting, follow up

139
ear examination:
History:
 you are FY2 in pediatric department.
 A 2 years old Angela has been brought by her mother
to the hospital with fever.
 She has been feeling unwell for the last 2 days
with a high fever of 39.
 Her mother has tried to give her PCM but the
temperature has been persistent.
 She has been crying while pulling her right ear.
 She has been taking less fluids than usual.
 She is up to date on her jabs, no rash or no other
symptoms.
 The mom is at work and works as a secretary at
anoffice ; she is worried about the high fever.
 baby is fit and well and not on any medicine
 She had never been admitted to the hospital.
 Everything was normal during pregnancy, delivery
was normal too.
Task:
Talk to the mother and address his concerns and discuss
management
Concerns:
What’s wrong with my baby?
examinations and investigations:

140
The child has been examined and the findings T39, BP
89/77, HR 146, pink TM on left ear,
Nose and throat arenormal.
Chest is clear.
Routine examination (blood), urine , ear discharge swab
test
Management:
 Greet and confirm identity
take focused history,
ask ODPARA of presenting complain,
explore fever, pulling of ear
Exclude red flags (meningitis, encephalopathy)
head to toe,
exclude dehydration(ask dehydration questions)
 Do ICE
 do ear examination in the mannikin and explain
the finding, summarize the condition its a viral infection
of ear
 Treatment: the baby is drowsy and dehydrated, do
symptomatic treatment, paracetamol, fluids
 Involve senior
 Advice: give her plenty of fluids and assure her
 Safety netting

141
DKA refusal
History:
 You are FY2 in emergency department
 35 year old lady has come to the hospital with
abdominal pain and vomiting

 She is a single mom. She has a wedding in 5 days.


Shehas to attend. She doesn’t want to be admitted and
want to go home.

 She was lying on hospital bed with IV fluids.


 she has DM type 1 for 5 years .
 She was on 20 units long acting insulin which she
takesin the morning.

 For the last 5 days she has been busy with work and
herchild so she did not take the insulin.

 no other symptoms, patient doesn’t want to stay in


thehospital, she needs to bring her child from nursery
too

Task:

142
Take focused history, discuss diagnosis and
management.

Concern:
What are they going to do for me in the hospital?
doctor I have my wedding needs preparations also
I need bring my daughter from the nursery

Examination and investigation:


take observations, examine tummy
Blood (RBS/ ESR, CRP/ cholesterol/ ABG/ KFT),
Urine (dipstick culture ketone bodies), chest X-ray for
infection,
Erect abdominal X-ray, RBS, ABG (metabolic acidosis)
Management:
 greet and confirm identity

explore presenting complaints and take full history of


diabetes

for how long,

medication,

if it’s controlled,

complications,

check ups,

any challenges, what happened today

143
exclude red flags

infection,

DESA,

MAFTOSA

 Do ICE,

 verbalize examination and investigation and explain


what is DKA; it’s a serious condition if not treated can
be life threatening

 tell the patient he needs admission, if patient


refuses

 use refusal approach

 Treatment in the hospital: insulin via drip, fluid to


reduce dehydration

 Address concern of the patient, ask for the reason


of refusal and give solutions, if she says she has
wedding preparations tell her can anyone give her a
hand for thepreparations , also ask for someone else to
bring her child

 Explain she needs to stay at the hospital now as


her condition might get worse which will require more
aggressive treatment

144
BBN
History:
 You are FY2 in vascular surgery
 A 60year old lady has undergone a aorto femoral
bypass surgery 1 hour back
 After the surgery , she was noted bleeding from the
drain , though operation went well
 She was given 6units of blood as she lost so much
blood
 She lives with her husband who is in the hospital , he
knows her wife had circulation problem and undergone
surgery , he was not acknowledge about her bleeding
 Her two children lives in Australia
 Husband claims something wrong has happened
during operation
Task:
Talk to him and address his concerns:
Concern:
What happened to my wife?
Why she bleed this much ?
any wrong with the operation?

145
Is she going to die?
Shall I call my children? They live in Australia
Management:
 Greet and confirm identity , and build good rapport
Ask about how much he knows about his wife?
Has anyone talk to him about her so far?
What operation she had and complications of it?
Ask about support system
 do breaking bad approach: explain the operation, and
why we did that , if the operation was not done she
could have lost her leg
 Address concern:
1. Post op bleeding is a common complication as it is a
complicated surgery , this complication happens
2. We are trying our best to control the bleeding , she was
given 6units of blood
3. its unlikely for any mistake but we are trying to fine out
the cause and fix it
4. We can not say if she is going to die, but we are trying
our best to save her
 Ask him for support as it is a life threatening
condition
 Address concerns , be more empathetic , give pause
and listen to him

146
PAP smear Result:
History:
 You are FY2 in obstetrics and gynaecology department
 A 40 years old lady had a pap smear done 2
weeks back and has come for the test result
 She is normally fit and well, no past medical history or
medication history, grandmother died in ovarian cancer
 She is sexually active, and practice safe sex, family
is complete with two children
 She doesn’t smoke or drink
 Now completely fine, no symptoms like fever, dysuria or
bleeding or discharge per vagina
 She is very curious to know about the result
Task:
Explain the test result and do management
Concern:
What is this change you are talking doctor, is it cancer?
Examination:
Take observation and head to toe
Management:
 Greet and confirm identity

147
 Build good rapport and paraphrase the scenario
Take history
general health
any symptoms bleeding/ discharge
tummy pain
MMA: none
sexual history: sexually active , practice safe sex
Family history; grand mom had cancer
DESA, smoking alcohol: no
 Do ICE
 verbalize examination
 explain the test result that there is some changes came
in the cervix which is benign but pre cancerous that
means it mightturn into cancer but human papilloma
virus came negative
 Emphasis on taking cervical vaccine
 The reason behind it is unknown, but there are some
risk factors
 Tell her to do routine 3 yearly check up if no symptoms
arise like bleeding , discharge , painful sexual
intercourse
 Safety netting

148

You might also like